Site Loader

Содержание

Конденсатор в цепи переменного тока

Рассмотрим процессы, протекающие в электрической цепи переменного тока с конденсатором. Если подключить конденсатор к источнику постоянного тока, то в цепи возникнет кратковременный импульс тока, который зарядит конденсатор до напряжения источника, а затем ток прекратится. Если заряженный конденсатор отключить от источника постоянного тока и соединить его обкладки с выводами лампы накаливания, то конденсатор будет разряжаться, при этом наблюдается кратковременная вспышка лампы.

При включении конденсатора в цепь переменного тока процесс его зарядки длится четверть периода. После достижения амплитудного значения напряжение между обкладками конденсатора уменьшается и конденсатор в течение четверти периода разряжается. В следующую четверть периода конденсатор вновь заряжается, но полярность напряжения на его обкладках изменяется на противоположную и т.

д. Процессы зарядки и разрядки конденсатора чередуются с периодом, равным периоду колебаний приложенного переменного напряжения.

Как и в цепи постоянного тока, через диэлектрик, разделяющий обкладки конденсатора, электрические заряды не проходят. Но в результате периодически повторяющихся процессов зарядки и разрядки конденсатора по проводам, соединенным с его выводами, течет переменный ток. Лампа накаливания, включенная последовательно с конденсатором в цепь переменного тока (рис. 6), кажется горящей непрерывно, так как человеческий глаз при высокой частоте колебаний силы тока не замечает периодического ослабления свечения нити лампы.

Рис. 6

Установим связь между амплитудой колебаний напряжения на обкладках конденсатора и амплитудой колебаний силы тока. При изменениях напряжения на обкладках конденсатора по гармоническому закону

,

заряд на его обкладках изменяется по закону:

.

Электрический ток в цепи возникает в результате изменения заряда конденсатора: i = q’. Поэтому колебания силы тока в цепи происходят по закону:

.

Следовательно, колебания напряжения на обкладках конденсатора в цепи переменного тока отстают по фазе от колебаний силы тока на π/2 или колебания силы тока опережают по фазе колебания напряжения на π/2

(рис. 7). Это означает, что в момент, когда конденсатор начинает заряжаться, сила тока максимальна, а напряжение равно нулю. После того как напряжение достигает максимума, сила тока становится равной нулю и т.д.

Рис. 7

Произведение является амплитудой колебаний силы тока:

.

Отношение амплитуды колебаний напряжения на конденсаторе к амплитуде колебаний силы тока называют емкостным сопротивлением конденсатора (обозначается ХC):

.

Связь между амплитудным значением силы тока и амплитудным значением напряжения по форме совпадает с выражением закона Ома для участка цепи постоянного тока, в котором вместо электрического сопротивления фигурирует емкостное сопротивление конденсатора:

.

Емкостное сопротивление конденсатора, как и индуктивное сопротивление катушки, не является постоянной величиной. Оно обратно пропорционально частоте переменного тока. Поэтому амплитуда колебаний силы тока в цепи конденсатора при постоянной амплитуде колебаний напряжения на конденсаторе возрастает прямо пропорционально частоте.

2 Задача

Тело падает с некоторой высоты. В момент падения на землю его скорость равна 30м/с.С какой высоты упало тело?

Дано:

V0 = 30м/c

q = 9,8 м/с2

h = ?

Решение:

Примем за точку отсчета высоты ту точку, откуда упало тело. В этой точке кинетическая энергия тела равна нулю, а потенциальная энергия равна mqh.

В момент падения тела на землю потенциальная энергия равна нулю, а кинетическая энергия равна

mV02 / 2. Согласно закону сохранения энергии получаем : mqh.= mV

02 / 2. Отсюда :h = V02/2q

h = (30)2/2×9,8=45,9(м)

Ответ: 45,9(м)

Билет №31

1 Переменный электрический ток. Катушка индуктивности в цепи переменного тока.

Переменным называется ток, периодически изменяющий свое направление и величину, причем среднее значение этого тока за период равно нулю. На рисунке 1 видно, что через определенный промежуток времени Т, называемый периодом изменения тока повторяются.

i

t

T

Рисунок 1 – Период переменного тока

Периодом переменного тока называется отрезок времени, в течение которого ток выполняет одно полное колебание (эту единицу обозначают буквой Т). Число полных колебаний за 1 с называется частотой тока и обозначается буквой f. Частота измеряется в герцах (Гц). В промышленности и быту большинства стран используют переменный ток с частотой 50 Гц.

Длительность периода измеряется в секундах. Число периодов в секунду называется частотой, следовательно, частота f=1/T. Частота измеряется в Герцах (Гц)=1Гц=1/с. Частота тока в электроэнергетических установках стандартизирована. В энергосистемах Росси и многих других стран промышленная частота ЭДС (тока) равна 50 Гц .

Значение переменной электрической величины в какой-нибудь момент времени называется, мгновенным значением и ее обозначают малыми буквами i, u, e .

ω=2πf,

где f- угловая частота.

Наибольшее из мгновенных значений ЭДС, напряжение, тока имеющее место в течение периода, называется амплитудным (Еm, Um, Im).

Для расчета цепей переменного тока пользуются понятием

действующего значения переменного тока (Е, U).

Действующее значение переменного тока равно значению такого эквивалентного постоянного тока, который, проходя через то же сопротивление, что и переменный ток, выделяет в нем за период переменного тока тоже количество тепла.

Действующее значение обозначают прописными буквами, то есть ток I, напряжение U, ЭДС – Е. На шкалах измерительных приборов наносят действующие значения.

2, аналогично U=Um/√2, E=Em/√2,

Стадии измерения переменной величины называют ФАЗОЙ.

е1 = Еmsin · (ω

t1)

е2 = Еmsin · (ωt2)

В этих выражениях угол (ωt + ψ) называется фазным углом или фазой. Углы ψ1 и ψ2, определяющие значение ЭДС в начальный момент времени (t = 0), называются начальными фазами.

Разность начальных фаз двух синусоидальных величин называется углом сдвига фаз или сдвигом фаз (рисунок 2)

φ = ψ1 — ψ2

e1 e2

ωt

Ψ2

Ψ1

φ

Катушка в цепи переменного тока

Индуктивность влияет на силу переменного тока в цепи. Это можно обнаружить с помощью простого опыта. Составим цепь из катушки большой индуктивности и лампы накаливания (рис. 3). С помощью переключателя можно присоединять эту цепь либо к источнику постоянного напряжения, либо к источнику переменного напряжения. При этом постоянное напряжение и действующее значение переменного напряжения должны быть одинаковы. Опыт показывает, что лампа светится ярче при постоянном напряжении. Следовательно, действующее значение силы тока в рассматриваемой цепи меньше силы постоянного тока.

Рис. 3

Объясняется это самоиндукцией. При подключении катушки к источнику постоянного напряжения сила тока в цепи нарастает постепенно. Возникающее при нарастании силы тока вихревое электрическое поле тормозит движение электронов. Лишь по прошествии некоторого времени сила тока достигает наибольшего (установившегося) значения, соответствующего данному постоянному напряжению. Если напряжение быстро меняется, то сила тока не будет успевать достигать тех установившихся значений, которые она приобрела бы с течением времени при постоянном напряжении, равном максимальному значению переменного напряжения. Следовательно, максимальное значение силы переменного тока (его амплитуда) ограничивается индуктивностью

L цепи и будет тем меньше, чем больше индуктивность и чем больше частота приложенного напряжения.

Докажем это математически. Пусть в цепь переменного тока включена идеальная катушка с электрическим сопротивлением провода, равным нулю (рис. 4). При изменениях силы тока по гармоническому закону

.

в катушке возникает ЭДС самоиндукции

,

где L – индуктивность катушки, ω – циклическая частота переменного тока.

Рис. 4

Так как электрическое сопротивление катушки равно нулю, то ЭДС самоиндукции в ней в любой момент времени равна по модулю и противоположна по знаку напряжению на концах катушки, созданному внешним генератором:

.

Следовательно, при изменении силы тока в катушке по гармоническому закону напряжение на ее концах изменяется тоже по гармоническому закону, но со сдвигом фазы:

.

Следовательно, колебания напряжения на катушке индуктивности опережают колебания силы тока на π/2, или, что то же самое, колебания силы тока отстают по фазе от колебаний напряжения на π/2.

В момент, когда напряжение на катушке достигает максимума, сила тока равна нулю (рис. 5). В момент, когда напряжение становится равным нулю, сила тока максимальна по модулю.

Рис. 5

Произведение является амплитудой колебаний напряжения на катушке:

.

Отношение амплитуды колебаний напряжения на катушке к амплитуде колебаний силы тока в ней называется индуктивным сопротивлением (обозначается XL):

.

Связь амплитуды колебаний напряжения на концах катушки с амплитудой колебаний силы тока в ней совпадает по форме с выражением закона Ома для участка цепи постоянного тока:

.

В отличие от электрического сопротивления проводника в цепи посто-янного тока, индуктивное сопротивление не является постоянной величиной, характеризующей данную катушку. Оно прямо пропорционально частоте переменного тока. Поэтому амплитуда колебаний силы тока в катушке при постоянном значении амплитуды колебаний напряжения должна убывать обратно пропорционально частоте. Постоянный ток вообще «не замечает» индуктивности катушки. При ω = 0 индуктивное сопротивление равно нулю (XL = 0).

Зависимость амплитуды колебаний силы тока в катушке от частоты приложенного напряжения можно наблюдать в опыте с генератором пере-менного напряжения, частоту которого можно изменять. Опыт показывает, что увеличение в два раза частоты переменного напряжения приводит к уменьшению в два раза амплитуды колебаний силы тока через катушку.

2 Задача

Луч света падает на поверхность воды под углом 40°. Под каким углом должен упасть луч на поверхность стекла, чтобы угол преломления оказался тем же, что и в первом случае?

Дано:

α = 40°;

n воды = 1,33;

ncт = 1,6;

β1 = ‹β2;

α2 _— ?

Решение:

Среда 1 – вода : n1 = sinα1 / sinβ1

Среда 2 – стекло : n2 = sinα2 / sinβ2

По условию задачи β1 = β2 , значит , sin β1= sin β2. Из первой формулы : sin β1 = sinα1/n1

Аналогично sinβ2= sinα2/n2

Приравнивая оба выражения, получаем : sinα1/n1 = sinα2/n2

Отсюда находим : sin α2 =n2× sinα1/n1; sinα2 = 1,6×sin40° / 1,33 = 0,77

По таблице синусов находим : α= 50°.

Ответ: α= 50°.

Билет №32

1 Устройство, принцип работы и назначение трансформатора

Трансформатор

Трансформатор — устройство, служащее для преобразования силы и напряжения переменного тока при неизменной частоте.

Он был изобретен П.Н.Яблочковым в 1878 г., а технический трансформатор впервые создал И.Ф.Усагин в 1882 г.

Работа трансформатора основана на явлении электромагнитной индукции. Простейший трансформатор (рис. 1) представляет собой две изолированные друг от друга катушки (обмотки), намотанные на общий замкнутый сердечник.

нагрузке.

Рисунок 1 — Включение трансформатора в цепь переменного тока.

В работу трансформатора положен принцип взаимной индукции (рисунок2). Когда по первичной обмотке течет переменный ток, то вокруг нее создается переменное магнитное поле, которое индуктирует ЭДС в каждой из витков вторичной обмотки.

Рисунок 2.Магнитная связь между катушками.

Таким образом, трансформатор может быть представлен только двумя катушками, расположенными рядом. Но для того, чтобы улучшить магнитную связь между катушками и уменьшить магнитный поток рассеяния (потоком рассеяния называется магнитный поток первичной обмотки, силовые линии которого замыкаются через воздух, не пронизывая вторичную обмотку) обе обмотки трансформатора помещаются в стальной сердечник. Благодаря сердечнику (иногда его называют магнитопроводом) подавляющее большинство силовых линий первичной обмотки пронизывает витки вторичной обмотки и участвуют в наведении ЭДС. На рисунке 3 изображена электрическая схема трансформатора т.к. определенная величина ЭДС индуктируется в каждом витке вторичной обмотки, а витки в ней соединены последователь, то ЭДС на зажимах вторичной обмотки прямо пропорциональна числу витков в ней.

Рисунок 3

По одной из обмоток (первичной) пропускается преобразуемый переменный ток, а вторичная обмотка соединяется с потребителем. Ток в первичной обмотке создает в сердечнике переменный магнитный поток, который возбуждает ЭДС самоиндукции в каждом витке первичной катушки (ΔΦ — изменение магнитного потока через один виток за время Δt). Этот же магнитный поток пронизывает витки вторичной катушки и создает в каждом ее витке ЭДС индукции Если первичная обмотка имеет N1 витков, а вторичная N2 витков, то в обмотках индуцируются (без учета потерь на рассеивание магнитного потока) соответственно электродвижущие силы а их отношение т.е. возникающие в катушках ЭДС индукции (самоиндукции) пропорциональны числу витков в них:

Отношение числа витков в первичной обмотке к числу витков во вторичной называют коэффициентом трансформации k .

Если N2 > N1(k < 1), то трансформатор называется повышающим, а если N2 < N1(k > 1)понижающим.

Коэффициент трансформации определяется обычно при холостом ходе трансформатора, т.е. при разомкнутой цепи вторичной обмотки. В этом случае в первичной обмотке проходит так называемый ток холостого хода, действующее значение которого Ix. На основании закона Ома для замкнутой цепи действующие значения напряжения U1, приложенного к первичной обмотке, ЭДС самоиндукции и сила тока Ix в первичной обмотке связаны между собой соотношением где R1 — активное сопротивление первичной обмотки. Знак минус обусловлен тем, что ЭДС согласно правилу Ленца противофазна U1. Трансформатор проектируется так, чтобы в отсутствие нагрузки потребляемый из сети ток был незначительным. Это достигается выбором малого активного сопротивления R1 и достаточно большого индуктивного сопротивления ωL. Для увеличения индуктивности катушки в нее вводят стальной сердечник и наматывают достаточно большое число витков N1. Тогда сила тока будет мала и величиной IxR1 можно пренебречь. Следовательно,

Цепь вторичной обмотки при холостом ходе разомкнута, вследствие чего в ней тока нет, и напряжение на зажимах вторичной обмотки равно индуцированной в ней ЭДС индукции Поэтому коэффициент трансформации можно найти, измерив напряжения на концах катушек при холостом режиме

При включении во вторичную цепь какой-либо нагрузки (рабочий ход трансформатора) в ней начинает проходить ток нагрузки I2 (переменный, такой же частоты). Ток I2 создает в сердечнике магнитный поток, направленный по правилу Ленца навстречу потоку первичной обмотки. В результате суммарный поток магнитной индукции в первичной катушке уменьшается, уменьшается и ЭДС а следовательно, сила тока увеличивается. Увеличение тока в первичной цепи приводит к увеличению магнитного потока, ЭДС индукции и силы тока во вторичной цепи. Но увеличение тока во вторичной цепи сопровождается увеличением тока самоиндукции и, следовательно, уменьшением магнитного потока (который только что возрастал). В конце концов при постоянной нагрузке устанавливаются определенные магнитный поток, ЭДС индукции во вторичной цепи, ток I1 в первичной цепи (I1 > Ix). Таким образом, увеличение тока I2 во вторичной цепи автоматически приводит к увеличению тока I1 в первичной цепи, т.е. трансформатор автоматически регулирует потребление энергии в зависимости от нагрузки во вторичной цепи.

При рабочем ходе трансформатора происходит непрерывная передача энергии из первичной цепи во вторичную. Мощность, потребляемая в первичной цепи а выделяемая на нагрузке Коэффициент полезного действия трансформатора

Не вся энергия, потребляемая от генератора, передается потребителю. При работе трансформатора имеются потери на нагревание обмоток трансформатора, на рассеивание магнитного потока в пространство, на вихревые токи Фуко (см. Закон электромагнитной индукции) в сердечнике и его перемагничивание. Для уменьшения этих потерь принимаются следующие меры: 1) обмотка низкого напряжения делается большего сечения, так как по ней проходит ток большей силы; 2) сердечник делают замкнутым, что уменьшает рассеивание магнитного потока; 3) сердечник делают из изолированных пластин для уменьшения токов Фуко и др. Благодаря этим мерам КПД современных трансформаторов достигает =95—99%, сдвиги фаз между колебаниями силы тока и напряжения близки к нулю

Если иногда можно пренебречь потерями в трансформаторе, т.е. считать η =100%, то это значит, увеличивая с помощью трансформатора напряжение, мы во столько же раз уменьшаем силу тока и наоборот.

При рабочем режиме трансформатора напряжения на его обмотках уже не будут равны ЭДС. Учитывая потери только на активных сопротивлениях, напряжения U1 и U2 можно рассчитать, исходя из закона Ома для замкнутой цепи.

По закону Ома для замкнутой первичной цепи трансформатора алгебраическая сумма подводимого к трансформатору напряжения U1 и возникающей ЭДС самоиндукции равна падению напряжения в цепи (на активном сопротивлении первичной обмотки R1):

Отсюда

Для подключенной нагрузки R роль источника тока выполняет вторичная обмотка, ЭДС в которой Она должна быть равна падению напряжения во вторичной цепи (на нагрузке R и на активном сопротивлении R2 вторичной обмотки):

но

Следовательно, Откуда

Передача электроэнергии

Электрическую энергию производят на электростанциях. Ее надо передать потребителям, часто находящимся очень далеко от станции. Для этого между станцией и потребителем строят линии электропередач (ЛЭП).

При передаче электроэнергии неизбежны потери, связанные с нагреванием проводов. Возникает проблема уменьшения этих потерь.

По закону Джоуля—Ленца количество теплоты, выделяемое в проводнике при прохождении тока, равно

Чтобы уменьшить потери в ЛЭП, необходимо, как видно из закона, уменьшить сопротивление R или силу тока в ней. Сопротивление проводов будет меньше, если уменьшить l, но длина определяется расстоянием, на которое надо передавать электроэнергию. Можно увеличить площадь поперечного сечения S. Но это ведет к перерасходу дорогостоящего цветного металла и возникновению трудностей при закреплении проводов на столбах. Поэтому такой способ снижения потерь практически невозможен.

Другой путь заключается в уменьшении силы тока в линии передачи. Но при данной мощности уменьшение силы тока возможно лишь при увеличении напряжения.

Таким образом, при передаче электроэнергии на большие расстояния необходимо пользоваться высоким напряжением. Так, электроэнергия Волжской ГЭС передается в Москву при напряжении 500 кВ, от Саяно-Шу-шенской ГЭС — при напряжении 750 кВ.

На электростанциях генераторы вырабатывают электрическую энергию при напряжениях, не превышающих 20 кВ. Поэтому на электростанциях устанавливают повышающие трансформаторы, а на месте потребления — понижающие трансформаторы. На рисунке 1 представлена блок-схема линии передачи переменного тока. Так как трансформаторы обладают большим индуктивным сопротивлением, которое приводит к сдвигу фаз между током и напряжением, то для увеличения коэффициента мощности в цепь включают конденсаторы.

Рис. 1

Потери при передаче постоянного тока были бы меньше примерно в полтора раза (нет потери на перемагничивание, потери на реактивном сопротивлении). Но пока нет способов трансформации постоянного тока. Делается попытка промышленной передачи постоянного тока высокого напряжения на большие расстояния, но трансформируется переменный ток, который затем при высоком напряжении выпрямляется с помощью полупроводниковых приборов. После передачи постоянный ток преобразуется в переменный (в инверторах), который затем снова трансформируется. Трудности преобразований тока в такой линии передачи не позволяют пока широко использовать этот экономичный метод передачи электроэнергии. На рисунке 2 показана блок-схема линии передачи постоянного тока.

В современной технике нашли широкое применение трансформаторы различных конструкций. В радиотехнических устройствах используются небольшие, маломощные трансформаторы, имеющие обычно несколько обмоток (понижающих или повышающих напряжение источника переменного тока). В электротехнике часто применяются так называемые трехфазные трансформаторы, предназначенные для одновременного повышения или понижения трех напряжений, сдвинутых по фазе относительно друг друга на углы 120°. Мощные трехфазные трансформаторы используются в линиях передач электроэнергии на большие расстояния. Передача электрической энергии от электростанций до больших городов или промышленных центров на расстояния тысяч километров является сложной научно-технической проблемой. Для уменьшения потерь на нагревания проводов необходимо уменьшить силу тока в линии передачи, и, следовательно, увеличить напряжение. Обычно линии электропередачи строятся в расчете на напряжение 400–500 кВ, при этом в линиях используется трехфазный ток частотой 50 Гц. На рис. 5.5.2 представлена схема линии передачи электроэнергии от электростанции до потребителя. Схема дает представление об использовании трансформаторов при передаче электроэнергии. Следует отметить, что при повышении напряжения в линиях передачи увеличиваются утечки энергии через воздух. В сырую погоду вблизи проводов линии может возникнуть ток называетмый коронный разряд, который можно обнаружить по характерному потрескиванию. Коэффициент полезного действия линии передач не превышает 90 %.

2

Рисунок 5.5.2. Условная схема высоковольтной линии передачи. Трансформаторы изменяют напряжение в нескольких точках линии. На схеме изображен только один из трех проводов высоковольтной линии.

2 Задача

Тепловоз массой 130 т приближается со скоростью 2 м/с к неподвижному составу массой 1170 т. С какой скоростью будет двигаться состав после сцепления с тепловозом?

Дано:

m1= 130 т = 1,3 ×103 кг

m2 = 1170 т = 11,7×103 кг

V1 = 2м/с

V2 = 0 м/с

m3= m1+ m2

Решение:

Согласно закону сохранения импульса проекции вектора полного импульса системы из тепловоза и состава на ось координат, направленную по вектору скорости, до сцепления и после сцепления одинаковы:

m1V1x +m2V2x = m3V3x

Так как состав был неподвижным , векторы скорости V1 тепловоза до сцепления и скорости V3 тепловоза вместе с составом после сцепления параллельны. Поэтому проекции векторов V1x и V3x можно заменить модулями этих векторов:

m1V1+ m2V2= m3 V3

Отсюда: V3= m1V1 + m2V2 / m3

V3 = 1,3 ×103 × 2+0 /1,3 ×103 +11,7 ×105=0,2 м/с

Ответ: 0,2 м/с

V3= ?

Скорость заряда конденсатора. Как ведет себя конденсатор в цепи переменного тока

Подключен к резистору, то ток и напряжение в цепи в любой точке временной диаграммы будут пропорциональны друг другу. Это означает, что кривые тока и напряжения будут достигать «пикового» значения одновременно. При этом мы говорим, что ток и напряжение находятся в фазе.

Рассмотрим теперь, как будет себя вести конденсатор в цепи переменного тока.

Если к источнику переменного напряжения подключен конденсатор, максимальное значение напряжения на нем будет пропорционально максимальному значению тока, протекающего в цепи. Однако пик волны синусоиды напряжения не будет наступать в то же самое время, что и максимум тока.

В этом примере мгновенное значение тока достигает своего максимального значения на четверть периода (90 эл.град.) раньше, чем это сделает напряжение. В таком случае говорят, что «ток опережает напряжение на 90◦».

В отличие от от ситуации в цепи постояннго тока, значение V/I здесь не является постоянным. Тем не менее, отношение V max/I max является весьма полезной величиной и в электротехнике называется емкостным сопротивлением (Хс) компонента. Поскольку эта величина по-прежнему отображает отношение напряжения к току, т.е. в физическом смысле является сопротивлением, ее единицей измерения является Ом. Значение Хс конденсатора зависит от его емкости (С) и частоты переменного тока (f).

Так как на конденсатор в цепи переменного тока подается среднеквадратичное значение напряжения, в этой цепи протекает такой же переменный ток, который ограничивается конденсатором. Это ограничение обусловлено конденсатора.


Поэтому значение тока в цепи, не содержащей никаких других компонентов, кроме конденсатора, определяется альтернативной версией Закона Ома

I RMS = U RMS / X C

Где U RMS — среднеквадратическое (действующее) значение напряжения. Обратите внимание, что X с заменяет величину R в версии закона Ома для

Теперь мы видим, что конденсатор в цепи переменного тока ведет себя совсем не так, как постоянный резистор, и ситуация здесь, соответственно, обстоит сложнее. Для того чтобы лучше понять процессы, происходящие в такой цепи, полезно ввести такое понятие, как вектор.

Основная идея вектора — это представление о том, что комплексное значение изменяющегося во времени сигнала может быть представлено ​​как произведение (которое не зависит от времени) и некоего комплексного сигнала, являющегося функцией времени.

Например, мы можем представить функцию A cos(2πνt + θ) просто как сложную постоянную A∙e jΘ .

Так как векторы представлены величиной (или модулем) и углом, то графически они представляются стрелкой (или вектором), вращающейся в плоскости XY.

С учетом того, что напряжение на конденсаторе «запаздывает» по отношению к току, представляющие их векторы расположены в комплексной плоскости так, как показано на рисунке выше. На этом рисунке векторы тока и напряжения вращаются в направлении, противоположном движению часовой стрелки.

В нашем примере ток на конденсаторе обусловлен его периодическим перезарядом. Поскольку конденсатор в цепи переменного тока обладает способностью периодически накапливать и сбрасывать электрический заряд, между ним и источником питания происходит постоянный обмен энергией, которая в электротехнике называется реактивной.

На рис. 4.11 показана цепь электрического генератора, содержащая конденсатор. После включения цепи вольтметр, включенный в цепь, покажет полное напряжение генератора. Стрелка амперметра установится на нуле — ток через изоляцию конденсатора протекать не может.

Но проследим внимательно за стрелкой амперметра при включении незаряженного конденсатора. Если амперметр достаточно чувствителен, а емкость конденсатора велика, то нетрудно обнаружить колебание стрелки: сразу после включения стрелка сойдет с нуля, а затем быстро вернется в исходное положение.

Рис. 4.11. Цепь электрического генератора, содержащая конденсатор

Этот опыт показывает, что при включении конденсатора (при его зарядке) в цепи протекал ток — в ней происходило передвижение зарядов: электроны с пластины, присоединенной к положительному полюсу источника, перешли на пластину, присоединенную к отрицательному полюсу.

Как только конденсатор зарядится, движение зарядов прекращается.

Отключая генератор и повторно замыкая его на конденсатор, мы уже не обнаружим движения стрелки: конденсатор остается заряженным, и при повторном включении движения зарядов в цепи не происходит.

Для того чтобы вновь наблюдать отклонение стрелки, нужно замыкать генератор на разряженный конденсатор. С этой целью, предварительно отключив генератор, замкнем пластины конденсатора проволокой, при этом между зажимами конденсатора и подносимой к ним проволокой проскочит искра, тем самым легко убедиться, что при разряде конденсатора в его цепи опять протекал ток.

Если замыкание проволокой произвести так, чтобы путь зарядов проходил через амперметр, то легко увидеть, что его стрелка кратковременно отклонится. Отклонение стрелки теперь должно происходить, конечно, в другую сторону.

После разряда конденсатора можно повторить первый опыт — стрелка амперметра вновь покажет, что в цепи конденсатора передвигаются электрические заряды (проходит ток).

Попытаемся вычислить ток, протекающий в проводах, присоединенных к конденсатору.

Если за промежуток времени напряжение конденсатора увеличивается на , то, значит, за это же время его заряд увеличится на

т. е. заряд конденсатора возрастает на произведение емкости и приращения напряжения.

Предположим, что напряжение на конденсаторе емкостью возросло на 50 В за время в одну десятую долю секунды . В таком случае за это же время заряд положительной пластины конденсатора увеличился на

Но для того чтобы такой заряд прошел по проводам за время с, нужно, чтобы по ним протекал средний ток

Заряд конденсатора через резистор. Представим себе, что генератор с постоянным напряжением замыкается через резистор с сопротивлением на незаряженный конденсатор емкостью (рис. 4.12, а).

В начальный момент, пока еще конденсатор не заряжен, его напряжение равно нулю.

Значит все напряжение источника приходится на сопротивление R. А это значит, что по закону Ома в цепи будет протекать ток

С течением времени, напротив, конденсатор зарядится, его напряжение будет равно напряжению генератора, в цепи не будет тока, на резисторе не будет никакого напряжения.


Рис. 4.12. а — заряд конденсатора С через резистор с сопротивлением Слева показана электрическая схема, на которой применено общепринятое изображение конденсатора, справа показано, как с течением времени нарастает напряжение на конденсаторе «с и как постепенно убывает ток г. Эти графики построены в предположении, что конденсатор емкостью 100 мкФ заряжается от источника постоянного напряжения 100 В через сопротивление 10 000 Ом. В этом случае заряд происходит очень медленно. Если бы емкость составила всего 1 мкФ, а сопротивление 1 Ом, все происходило бы в миллион раз скорее. Для того чтобы приведенные графики оказались пригодными и для второго случая, нужно считать, что время выражено не в секундах, а в миллионных долях секунды (в общем случае при любых R и С указанные на графике значения времени следует умножить на произведение С и Я). Если напряжение источника остается 100 В, то значения тока должны быть увеличены в 10 000 раз. Например, в начальный момент будет протекать ток не 10 мА, а 100 А. Длительность и характер процесса не зависят от напряжения источника; б — разряд конденсатора С через резистор сопротивлением R. Слева показана электрическая схема. После заряда конденсатор отключается. Справа показано, как изменяются ток и напряжение конденсатора с течением времени. Графики построены для случая . Уменьшение емкости и сопротивления до значений и 1 Ом увеличило бы скорость разряда в миллион раз. Начальное; значение тока (при неизменности начального напряжения) при этом возросло бы в 10 000 раз и составило бы 100 А вместо 10 мА. При других значениях R и С время, показанное на графике, нужно умножить на произведение

При этом заряд конденсатора должен быть равен

Поставим такой вопрос: как скоро заряд в одну сотую кулона может быть сообщен конденсатору?

Если бы в цепи ток не уменьшался, а оставался равным т. е. 10 мА, то для этого потребовалось бы время, равное всего лишь 1 с:

Но сообразим, может ли долго протекать такой ток, как Если бы такой ток протекал четверть секунды, он уже сообщил бы конденсатору четверть полного заряда, а значит, поднял бы его напряжение до четверти от полных 100 В.

Но когда напряжение конденсатора возрастет до 25 В, ток должен уменьшиться до 7,5 мА. В самом деле, если напряжение генератора 100 В, а напряжение на конденсаторе 25 В, то разность между ними приходится на резистор.

Опять же по закону Ома

Но такой ток будет заряжать конденсатор медленнее, чем его заряжал ток в 10 мА.

Из приведенного рассуждения ясно, что:

нарастание напряжения на конденсаторе будет происходить, постепенно замедляясь;

ток, достигнув наибольшего значения в начальный момент, потом постепенно уменьшится;

чем больше емкость (больше заряд) и чем больше сопротивление цепи, тем медленнее происходит заряд конденсатора.

Разряд конденсатора на резистор. Если отключить генератор и через резистор с сопротивлением R замкнуть пластины конденсатора, начнется процесс его разряда. На рис. 4.12, б приведены кривые тока и напряжения конденсатора при его разряде.

Энергия электрического поля в конденсаторе. Заряженный конденсатор обладает определенным запасом энергии, заключенной в его электрическом поле.

Об этом можно судить по тому, что заряженный конденсатор, отключенный от сети, способен некоторое время поддерживать электрический ток — об этом можно судить и по искре, наблюдаемой при разряде конденсаторов.

Энергия, заключенная в конденсаторе, подводится к нему в то время, когда он заряжается от генератора. В самом деле, во время его заряда в цепи течет ток и к его зажимам приложено напряжение, а это значит, что ему сообщается энергия. Полное количество энергии, запасенной конденсатором, может быть выражено формулой

Энергия равна половине квадрата напряжения, умноженного на емкость.

Если напряжение выражено в вольтах, а емкость — в фарадах, то энергия окажется выраженной в джоулях.

Так, энергия, запасенная в конденсаторе емкостью 100 мкФ при напряжении 1000 В,

Это, конечно, не очень большая энергия (такая энергия поглощается лампочкой 50 Вт за каждую секунду). Но если конденсатор быстро разряжается (скажем, за одну тысячную долю секунды), то мощность происходящего разряда энергии, конечно, очень велика:

Поэтому понятно, что при разряде большого конденсатора звук похож на выстрел.

Быстрым разрядом энергии, запасенной в конденсаторе, иногда пользуются для сварки маленьких металлических изделий.

При разряде конденсатора на резистор энергия, заключавшаяся в электрическом конденсаторе, переходит в тепло нагреваемого резистора.

Применение конденсаторов. Применения конденсаторов в электротехнике очень разнообразны.

Рассмотрим здесь некоторые из них.

1. Конденсаторы широко применяют для целей изоляции двух цепей по постоянному напряжению при сохранении связи между ними на переменном токе. Конденсаторы изолируют постоянное напряжение, не пропуская постоянный ток. В то же время малейшее изменение напряжения изменяет их заряд и, следовательно, пропускает через них соответствующий переменный ток (рис. 4.13).

Рис. 4.13. На входе схемы между точками а и б приложено постоянное напряжение и маленькое, изменяющееся во времени напряжение — его форма Соответствует передаваемому сигналу. Конденсатор не пропускает постоянный ток (соответствующий ). Маленькое изменяющееся напряжение А и меняет заряд конденсатора. Протекающий зарядный ток создает падение напряжения на большом сопротивлении цепи. Это падение напряжения очень близко к значению переменного напряжения Таким образом, напряжение на выходе схемы между точками в и г приблизительно равно

2. На свойствах конденсатора пропускать ток под действием изменяющегося напряжения и не пропускать ток под действием постоянного напряжения основаны сглаживающие устройства (фильтры, не пропускающие переменное напряжение). На рис. 4.14 показано такое устройство — переменный ток проходит через первый резистор и конденсатор, но благодаря большой емкости конденсатора колебание напряжения на нем очень мало. На выходе схемы напряжение сглажено — оно близко к постоянному.

Еще более сильное сглаживание можно получить, включая вместо резисторов индуктивные катушки L.


Рис. 4.14. Сглаживающее устройство, содержащее R и С. Колебания напряжения на входе схемы не передаются на выход. Напряжение на выходе близко к постоянному

Как было показано в гл. 2, при протекании изменяющегося тока в них наводится ЭДС, препятствующая колебаниям тока. Такое сглаживающее устройство показано на рис. 4.15.

3. На рис. 4.16 схематически показано устройство для зажигания горючей смеси в цилиндрах автомобильного двигателя.


Рис. 4.15. Сглаживающее устройство, содержащее L и С. На вход подано напряжение, заметно колеблющееся во времени. Напряжение на нагрузке почти постоянно

Ток от батареи проходит через первичную обмотку катушки. В нужный момент он прерывается специальными подвижными контактами. Быстрое изменение тока наводит ЭДС взаимоиндукции во вторичной обмотке катушки. Число витков вторичной обмотки очень велико, и разрыв тока производится быстро. Поэтому ЭДС, наводимая во вторичной обмотке, может достигать 10-12 тыс. В. При таком напряжении происходит искровой разряд между электродами «свечи», воспламеняющей рабочую смесь в цилиндре. Прерывание контакта происходит очень часто: так, в четырехцилиндровом двигателе один разрыв контактов происходит за каждый оборот двигателя.

На схеме на рис. 4.16 показан конденсатор, присоединенный к зажимам прерывателя.

Объясним его назначение.

При отсутствии конденсатора разрыв цепи сопровождался бы образованием искры между контактами прерывателя.


Рис. 4.16. Схема цепи, служащей для электрического зажигания горючей смеси в цилиндрах автомобильного двигателя: — прерыватель. Внизу показан разрез цилиндра с поршнем, над которым смесь воздуха с бензином воспламеняется электрической искрой, проскакивающей между электродами свечи

Не говоря уже о том, что часто появляющаяся искра быстро привела бы к износу контактов, наличие искры препятствует резкому разрыву тока: ток, после того как контакты разойдутся, еще остается замкнутым через искру и лишь постепенно спадает до нуля.

Если между контактами прерывателя включен конденсатор (как это показано на рис. 4.16), картина будет иной. Когда контакты начинают расходиться, цепь тока не разрывается — ток замыкается через еще не заряженный конденсатор. Но конденсатор быстро заряжается, и дальнейшее протекание тока оказывается невозможным.

Напряжение на заряженном конденсаторе может намного превысить 12 В, так как уменьшение тока в первичной обмотке катушки наводит в ней большую ЭДС самоиндукции.

Несмотря на это между контактами прерывателя искра уже не возникает, так как к этому моменту контакты прерывателя успевают достаточно далеко отойти один от другого.

Когда контакты прерывателя вновь замкнутся, конденсатор быстро разрядится и будет готов к работе при новом разрыве контактов.

Таким образом, конденсатор предохраняет контакты от обгорания и улучшает работу системы зажигания.

На схеме на рис. 4.16 рядом с конденсатором может быть включено добавочное сопротивление. Его назначение станет ясным после того, как мы рассмотрим электрические колебания в системе индуктивность — конденсатор.

Рис. 4.17. Разряд конденсатора на индуктивность. В такой цепи возникают электрические колебания (см. рис., 4.18)

4. Одно из очень важных применений конденсаторы находят в цепях переменного тока (улучшение «косинуса фи»). Оно рассмотрено в гл. 6.

О применении конденсаторов в колебательных контурах генераторов рассказано в гл. 8.

Эти применения конденсаторов основаны на электрических колебаниях в системе LC (индуктивность и емкость).

Разряд конденсатора на индуктивность. Электрические колебания. Рассмотрим, что произойдет, если заряженный конденсатор замкнуть на катушку, обладающую индуктивностью и очень малым сопротивлением (рис. 4.17).

Возьмем конденсатор С, заряженный до напряжения в его электрическом поле при этом запасена энергия

Замкнем конденсатор на индуктивную катушку. Очевидно, что конденсатор начнет разряжаться. Однако благодаря возникающей ЭДС самоиндукции ток в катушке возрастает постепенно (§ 2.16 и 2.18). Ток первоначально был равен нулю, постепенно он возрастает. По мере протекания тока разряжается конденсатор; его напряжение при этом уменьшается.

Но мы знаем, что скорость нарастания тока — или вообще скорость изменения тока — в индуктивности пропорциональна приложенному к ней напряжению (внимательно рассмотрите, если нужно, § 2.16).

По мере уменьшения напряжения на конденсаторе уменьшеется скорость нарастания тока.

Мы сказали, что уменьшается скорость нарастания тока, но это вовсе не значит, что уменьшается сам ток.

Рис. 4.18. Изменения напряжения на конденсаторе и разрядного тока в цепи, изображенной на рис. 4.17. Приведенные здесь значения тока и напряжения соответствуют разряду конденсатора емкостью С=4мкФ, предварительно заряженного до напряжения . Индуктивность катушки L = 1,6 мГн. Этим данным соответствует период

Действительно, рассмотрим графики напряжения на конденсаторе и тока, представленные на рис. 4.18.

Сначал ток был равен нулю, но возрастал он очень быстро (это видно по крутизне подъема кривой линии, изображающей зависимость тока от времени). В конце разряда конденсатора, когда его напряжение стало равным нулю, ток перестал возрастать — он достиг наибольшего значения и уже не возрастает дальше.

Мы можем всё сказанное выразить таким уравнением:

Напряжение на конденсаторе всегда равное напряжению на индуктивности, равно скорости нарастания тока умноженной на индуктивность L.

Конденсатор разрядился.

Энергия, заключенная в электрическом поле конденсатора, покинула конденсатор. Но куда она перешла?

В случае разряда конденсатора на сопротивление энергия перешла в тепло нагретого сопротивления. Но в рассматриваемом сейчас примере сопротивление цепи ничтожно (мы пренебрегли им вовсе). Где же теперь энергия, заключавшаяся в конденсаторе?

Энергия перешла из электрического поля конденсатора в магнитное поле индуктивности.

В самом деле, в начале процесса тока в индуктивности не было; когда ток в индуктивности достиг величины в ее магнитном поле появилась энергия

На основании закона сохранения энергии нетрудно найти то наибольшее значение которое достигается током в момент равенства нулю напряжения на конденсаторе.

В этот момент в конденсаторе нет энергии, значит, вся первоначально запасенная в нем энергия перешла в энергию магнитного поля. Приравнивая их выражения, находим

Очевидно, что в любой момент времени, когда напряжение на конденсаторе меньше, чем а ток меньше, чем общая энергия равна сумме энергий электрического и магнитного полей:

Итак, мы объяснили, что происходит за промежуток времени, понадобившийся для полного разряда конденсатора.

На рис. 4.18 этому соответствуют кривые тока и напряжения, относящиеся к промежутку, обозначенному цифрой I (время от 0 до 125 мкс).

Но дело на этом не кончается. Хотя конденсатор разрядился полностью, в цепи протекает большой ток. Этот ток не может сразу исчезнуть, так как его существование связано с энергией магнитного поля.

Этот ток продолжает протекать в цепи и перезаряжает конденсатор: он продолжает уносить электроны с отрицательных пластин и переносить их на пластины положительные, точнее — переносить с пластин, которые были отрицательными, на пластины, которые были положительными. Знак заряда на пластинах теперь изменяется.

На конденсаторе появляется напряжение, препятствующее дальнейшему протеканию тока, и ток постепенно начинает уменьшаться.

К концу промежутка времени, обозначенного цифрой II (к моменту времени 250 мкс), ток спадает до нуля. Но к этому моменту конденсатор опять окажется полностью заряженным; вся энергия, перешедшая в магнитное поле, теперь вновь превратилась в энергию электрического поля.

Ток равен нулю. Конденсатор имеет такое же напряжение, как вначале (только другого знака). Все начинается снова, так, как было рассказано: конденсатор начинает разряжаться, ток начинает возрастать и т. д.

Разница только в знаке напряжения на конденсаторе и соответственно в направлении тока: ток остается отрицательным в течение промежутков времени, обозначенных цифрами III и IV.

В конце промежутка IV (т. е. после того как пройдет 500 мкс) все вернется к исходному состоянию — конденсатор заряжен положительно и тока нет.

Начиная с этого момента все повторяется сначала.

Рассмотренная картина и представляет собой электрические колебания в цепи LC.

Время, требующееся на то, чтобы после начала разряда все вернулось к исходному состоянию, называется периодом (Т).

При значениях емкости и индуктивности, для которых построены графики на рис. 4.18, один период составляет 500 мкс. Чем больше индуктивность и емкость, тем больше период колебаний.

Связь между этими тремя величинами выражается равенством

Рассмотренные колебания называют свободными (в отличие от вынужденных), так как они происходят при отсутствии постороннего источника энергии, который мог бы заставить изменяться напряжение по какому-либо другому закону.

Такие колебания будут рассмотрены ниже, в.гл. 5 и 6. Там будет показано следующее: один источник (генератор) дает напряжение, изменяющееся по закону, подобному показанному на рис. 4.18, и если к источнику подключена катушка индуктивности, то в ней будет протекать ток

Это равенство имеет тот же смысл, что и равенство (А).

Небольшой приведенный здесь расчет показывает, до какой степени электрику нужно знать математику и иметь сноровку в проведении алгебраических действий.

Мы рассмотрели колебания, происходящие при разряде конденсатора, пренебрегая сопротивлением цепи. На самом деле в любом колебательном контуре сопротивление нельзя считать равным нулю.

Наличие небольшого сопротивления цепи приводит к постепенному затуханию колебаний, так как в сопротивлении происходит рассеяние энергии электромагнитного поля — она превращается в тепло в соответствии с законом Джоуля — Ленца.

Рис. 4.19. Затухающий колебательный разряд. Приведенный график напряжения на конденсаторе соответствует данным: , начальное напряжение на конденсаторе .

Поэтому каждый раз, когда вся энергия вновь сосредоточивается в электрическом поле конденсатора, напряжение на конденсаторе оказывается меньше:

На рис. 4.19 показана кривая напряжения на конденсаторе в цепи RLC (т. е. в цепи, содержащей кроме индуктивности и емкости также и сопротивление).

При достаточно большом сопротивлении в цепи колебания вообще не возникают. Разряд конденсатора происходит, как говорят, апериодически. Такой разряд показан на рис. 4.20. Разряд может быть сделан апёриодическйм и посредством подключения сопротивления параллельно конденсатору.

Понятие о разнообразных применениях колебательной системы (колебательного контура) будет дано в гл. 6 и 8.

Рис. 4.20. Апериодический разряд конденсатора. На графике изображены напряжения и ток в цепи конденсатора при тех же индуктивности и емкости (L = 1,6 МГн, С=4 мкФ) и при сопротивлении цепи, равном 64 Ом

Сейчас мы ограничимся указанием на то, что наличие конденсатора между контактами прерывателя в автомобиле (рис. 4.16) может служить источником колебаний, мешающих радиоприему. Эти колебания могут «гаситься», если ввести добавочный резистор (в соответствии со схемой на рис. 4.20).

Details 16 April 2017

Господа, в сегодняшней статье я хотел бы рассмотреть такой интересный вопрос, как конденсатор в цепи переменного тока . Эта тема весьма важна в электричестве, поскольку на практике конденсаторы повсеместно присутствуют в цепях с переменным током и, в связи с этим, весьма полезно иметь четкое представление, по каким законам изменяются в этом случае сигналы. Эти законы мы сегодня и рассмотрим, а в конце решим одну практическую задачу определения тока через конденсатор.

Господа, сейчас для нас наиболее интересным моментом является то, как связаны между собой напряжение на конденсаторе и ток через конденсатор для случая, когда конденсатор находится в цепи переменного сигнала.

Почему сразу переменного? Да просто потому, что конденсатор в цепи постоянного тока ничем не примечателен. Через него течет ток только в первый момент, пока конденсатор разряжен. Потом конденсатор заряжается и все, тока нет (да-да, слышу, уже начали кричать, что заряд конденсатора теоретически длится бесконечно долгое время, да еще у него может быть сопротивление утечки, но пока что мы этим пренебрегаем). Заряженный конденсатор для постоянного тока – это как разрыв цепи . Когда же у нас случай переменного тока – тут все намного интереснее. Оказывается, в этом случае через конденсатор может протекать ток и конденсатор в этом случае как бы эквивалентен резистору с некоторым вполне определенным сопротивлением (если пока забить забыть про всякие там сдвиги фазы, об этом ниже). Нам надо каким-нибудь образом получить связь между током и напряжением на конденсаторе.

Пока мы будем исходить из того, что в цепи переменного тока находится только конденсатор и все. Без каких-либо других компонентов типа резисторов или индуктивностей. Напомню, что в случае, когда у нас в цепи находится исключительно одни только резисторы, подобная задача решается очень просто: ток и напряжения оказываются связанными между собой через закон Ома . Мы про это уже не один раз говорили. Там все очень просто: делим напряжение на сопротивление и получаем ток. А как же быть в случае конденсатора? Ведь конденсатор-то это не резистор. Там совсем иная физика протекания процессов, поэтому вот так вот с наскока не получится просто связать между собой ток и напряжение. Тем не менее, сделать это надо, поэтому давайте попробуем порассуждать.

Сперва давайте вернемся назад. Далеко назад. Даже очень далеко. К самой-самой первой моей статье на этом сайте. Старожилы должно быть помнят, что это была статья про силу тока . Вот в этой самой статье было одно интересное выражение, которое связывало между собой силу тока и заряд, протекающий через сечение проводника. Вот это самое выражение

Кто-нибудь может возразить, что в той статье про силу тока запись была через Δq и Δt – некоторые весьма малые величины заряда и времени, за которое этот заряд проходит через сечение проводника. Однако здесь мы будем применять запись через dq и dt – через дифференциалы. Такое представление нам потребуется в дальнейшем. Если не лезть глубоко в дебри матана, то по сути dq и dt здесь особо ничем не отличаются от Δq и Δt . Безусловно, глубоко сведущие в высшей математике люди могут поспорить с этим утверждением, но да сейчас я не хочу концентрировать внимание на данных вещах.

Итак, выражение для силы тока мы вспомнили. Давайте теперь вспомним, как связаны между собой емкость конденсатора С , заряд q , который он в себе накопил, и напряжение U на конденсаторе, которое при этом образовалось. Ну, мы же помним, что если конденсатор накопил в себе какой-то заряд, то на его обкладках неизбежно возникнет напряжение. Про это все мы тоже говорили раньше, вот в этой вот статье . Нам будет нужна вот эта формула, которая как раз и связывает заряд с напряжением

Давайте-ка выразим из этой формулы заряд конденсатора:


А теперь есть очень большой соблазн подставить это выражение для заряда конденсатора в предыдущую формулу для силы тока. Приглядитесь-ка повнимательнее – у нас ведь тогда окажутся связанными между собой сила тока, емкость конденсатора и напряжение на конденсаторе! Сделаем эту подстановку без промедлений:


Емкость конденсатора у нас является величиной постоянной . Она определяется исключительно самим конденсатором , его внутренним устройством, типом диэлектрика и всем таким прочим. Про все это подробно мы говорили в одной из прошлых статей . Следовательно, емкость С конденсатора, поскольку это константа, можно смело вынести за знак дифференциала (такие вот правила работы с этими самыми дифференциалами). А вот с напряжением U нельзя так поступить! Напряжение на конденсаторе будет изменяться со временем . Почему это происходит? Ответ элементарный: по мере протекания тока на обкладках конденсатора, очевидно, заряд будет изменяться. А изменение заряда непременно приведет к изменению напряжения на конденсаторе. Поэтому напряжение можно рассматривать как некоторую функцию времени и его нельзя выносить из-под дифференциала. Итак, проведя оговоренные выше преобразования, получаем вот такую вот запись:


Господа, спешу вас поздравить – только что мы получили полезнейшее выражение, которое связывает между собой напряжение, приложенное к конденсатору, и ток, который течет через него. Таким образом, если мы знаем закон изменения напряжения, мы легко сможем найти закон изменения тока через конденсатор путем простого нахождения производной.

А как быть в обратном случае? Допустим, нам известен закон изменения тока через конденсатор и мы хотим найти закон изменения напряжения на нем. Читатели, сведущие в математике, наверняка уже догадались, что для решения этой задачи достаточно просто проинтегрировать написанное выше выражение. То есть, результат будет выглядеть как-то так:


По сути оба этих выражений про одно и тоже. Просто первое применяется в случае, когда нам известен закон изменения напряжения на конденсаторе и мы хотим найти закон изменения тока через него, а второе – когда нам известно, каким образом меняется ток через конденсатор и мы хотим найти закон изменения напряжения. Для лучшего запоминания всего этого дела, господа, я приготовил для вас поясняющую картинку. Она изображена на рисунке 1.


Рисунок 1 – Поясняющая картинка

На ней, по сути, в сжатой форме изображены выводы, которые хорошо бы запомнить.

Господа, обратите внимание – полученные выражения справедливы для любого закона изменения тока и напряжения. Здесь не обязательно должен быть синус, косинус, меандр или что-то другое. Если у вас есть какой-то совершенно произвольный, пусть даже совершенно дикий, не описанный ни в какой литературе, закон изменения напряжения U(t) , поданного на конденсатор, вы, путем его дифференцирования можете определить закон изменения тока через конденсатор. И аналогично если вы знаете закон изменения тока через конденсатор I(t) то, найдя интеграл, сможете найти, каким же образом будет меняться напряжение.

Итак, мы выяснили как связать между собой ток и напряжение для абсолютно любых, даже самых безумных вариантов их изменения. Но не менее интересны и некоторые частные случаи. Например, случай успевшего уже нам всем полюбиться синусоидального тока. Давайте теперь разбираться с ним.

Пусть напряжение на конденсаторе емкостью C изменяется по закону синуса вот таким вот образом

Какая физическая величина стоит за каждой буковкой в этом выражении мы подробно разбирали чуть раньше . Как же в таком случае будет меняться ток? Используя уже полученные знания, давайте просто тупо подставим это выражение в нашу общую формулу и найдем производную

Или можно записать вот так

Господа, хочу вам напомнить, что синус ведь только тем и отличается от косинуса, что один сдвинут относительно другого по фазе на 90 градусов. Ну, или, если выражаться на языке математики, то . Не понятно, откуда взялось это выражение? Погуглите формулы приведения . Штука полезная, знать не помешает. А еще лучше, если вы хорошо знакомы с тригонометрическим кругом , на нем все это видно очень наглядно.

Господа, отмечу сразу один момент. В своих статьях я не буду рассказывать про правила нахождения производных и взятия интегралов. Надеюсь, хотя бы общее понимание этих моментов у вас есть. Однако даже если вы не знаете, как это делать, я буду стараться излагать материал таким образом, чтобы суть вещей была понятна и без этих промежуточных выкладок. Итак, сейчас мы получили немаловажный вывод – если напряжение на конденсаторе изменяется по закону синуса, то ток через него будет изменяться по закону косинуса. То есть ток и напряжение на конденсаторе сдвинуты друг относительно друга по фазе на 90 градусов. Кроме того, мы можем относительно легко найти и амплитудное значение тока (это множители, которые стоят перед синусом). Ну то есть тот пик, тот максимум, которого ток достигает. Как видим, оно зависит от емкости C конденсатора, амплитуды приложенного к нему напряжения U m и частоты ω . То есть чем больше приложенное напряжение, чем больше емкость конденсатора и чем больше частота изменения напряжения, тем большей амплитуды достигает ток через конденсатор. Давайте построим график, изобразив на одном поле ток через конденсатор и напряжение на конденсаторе. Пока без конкретных цифр, просто покажем качественный характер. Этот график представлен на рисунке 2 (картинка кликабельна).


Рисунок 2 – Ток через конденсатор и напряжение на конденсаторе

На рисунке 2 синий график – это синусоидальный ток через конденсатор, а красный – синусоидальное напряжение на конденсаторе. По этому рисунку как раз очень хорошо видно, что ток опережает напряжение (пики синусоиды тока находятся левее соответствующих пиков синусоиды напряжения, то есть наступают раньше ).

Давайте теперь проделаем работу наоборот. Пусть нам известен закон изменения тока I(t) через конденсатор емкостью C . И закон этот пусть тоже будет синусоидальным

Давайте определим, как в таком случае будет меняться напряжение на конденсаторе. Воспользуемся нашей общей формулой с интегральчиком:

По абсолютнейшей аналогии с уже написанными выкладками, напряжение можно представить вот таким вот образом

Здесь мы снова воспользовались интересными сведениями из тригонометрии, что . И снова формулы приведения придут вам на помощь, если не понятно, почему получилось именно так.

Какой же вывод мы можем сделать из данных расчетов? А вывод все тот же самый, какой уже был сделан: ток через конденсатор и напряжение на конденсаторе сдвинуты по фазе друг относительно друга на 90 градусов. Более того, они не просто так сдвинуты. Ток опережает напряжение. Почему это так? Какая за этим стоит физика процесса? Давайте разберемся.

Представим, что незаряженный конденсатор мы подсоединили к источнику напряжения. В первый момент никаких зарядов в конденсаторе вообще нет: он же разряжен. А раз нет зарядов, то нет и напряжения. Зато ток есть, он возникает сразу при подсоединении конденсатора к источнику. Замечаете, господа? Напряжения еще нет (оно не успело нарасти), а ток уже есть . И кроме того, в этот самый момент подключения ток в цепи максимален (разряженный конденсатор ведь по сути эквивалентен короткому замыканию цепи). Вот вам и отставание напряжения от тока. По мере протекания тока, на обкладках конденсатора начинает накапливаться заряд, то есть напряжение начинает расти а ток постепенно уменьшаться. И через некоторое время накопится столько заряда на обкладках, что напряжение на конденсаторе сравняется с напряжением источника и ток в цепи совсем прекратится.

Теперь давайте этот самый заряженный конденсатор отцепим от источника и закоротим накоротко. Что получим? А практически то же самое. В самый первый момент ток будет максимален, а напряжение на конденсаторе останется таким же, какое оно и было без изменений. То есть снова ток впереди, а напряжение изменяется вслед за ним. По мере протекания тока напряжение начнет постепенно уменьшаться и когда ток совсем прекратится, оно тоже станет равным нулю.

Для лучшего понимания физики протекающих процессов можно в который раз уже использовать водопроводную аналогию . Представим себе, что заряженный конденсатор – это некоторый бачок, полный воды. У этого бачка есть внизу краник, через который можно спустить воду. Давайте этот краник откроем. Как только мы его откроем, вода потечет сразу же. А давление в бачке будет падать постепенно, по мере того, как вода будет вытекать. То есть, грубо говоря, ручеек воды из краника опережает изменение давления, подобно тому, как ток в конденсаторе опережает изменение напряжения на нем.

Подобные рассуждения можно провести и для синусоидального сигнала, когда ток и напряжения меняются по закону синуса, да и вообще для любого. Суть, надеюсь, понятна.

Давайте проведем небольшой практический расчет переменного тока через конденсатор и построим графики.

Пусть у нас имеется источник синусоидального напряжения, действующее значение равно 220 В , а частота 50 Гц . Ну, то есть все ровно так же, как у нас в розетках. К этому напряжению подключают конденсатор емкостью 1 мкФ . Например, пленочный конденсатор К73-17 , рассчитанный на максимальное напряжение 400 В (а на меньшее напряжение конденсаторы ни в коем случае нельзя подключать в сети 220 В), выпускается с емкостью 1 мкФ. Чтобы вы имели представление, с чем мы имеем дело, на рисунке 3 я разместил фотографию этого зверька (спасибо Diamond за фото )


Рисунок 3 – Ищем ток через этот конденсатор

Требуется определить, какая амплитуда тока будет протекать через этот конденсатор и построить графики тока и напряжения.

Сперва нам надо записать закон изменения напряжения в розетке. Если вы помните, амплитудное значение напряжения в этом случае равно около 311 В. Почему это так, откуда получилось, и как записать закон изменения напряжения в розетке, можно прочитать вот в этой статье . Мы же сразу приведем результат. Итак, напряжение в розетке будет изменяться по закону

Теперь мы можем воспользоваться полученной ранее формулой, которая свяжет напряжение в розетке с током через конденсатор. Выглядеть результат будет так

Мы просто подставили в общую формулу емкость конденсатора, заданную в условии, амплитудное значение напряжения и круговую частоту напряжения сети. В результате после перемножения всех множителей имеем вот такой вот закон изменения тока

Вот так вот, господа. Получается, что амплитудное значение тока через конденсатор чуть меньше 100 мА. Много это или мало? Вопрос нельзя назвать корректным. По меркам промышленной техники, где фигурируют сотни ампер тока, очень мало. Да и для бытовых приборов, где десятки ампер не редкость – тоже. Однако для человека даже такой ток представляет большую опасность! Отсюда следует вывод, что хвататься за такой конденсатор, подключенный к сети 220 В не следует . Однако на этом принципе возможно изготовление так называемых источников питания с гасящим конденсатором. Ну да это тема для отдельной статьи и здесь мы не будем ее затрагивать.

Все это хорошо, но мы чуть не забыли про графики, которые должны построить. Надо срочно исправляться! Итак, они представлены на рисунке 4 и рисунке 5. На рисунке 4 вы можете наблюдать график напряжения в розетке, а на рисунке 5 – закон изменения тока через конденсатор, включенный в такую розетку.


Рисунок 4 – График напряжения в розетке

Рисунок 5 – График тока через конденсатор

Как мы можем видеть из этих рисунков, ток и напряжение сдвинуты на 90 градусов, как и должно быть. И, возможно, у читателя возникла мысль – если через конденсатор течет ток и на нем падает какое-то напряжение, вероятно, на нем должна выделяться и некоторая мощность . Однако спешу предупредить вас – для конденсатора дело обстоит совершенно не так . Если рассматривать идеальный конденсатор, то мощность на нем не будет вообще выделяться, даже при протекании тока и падении на нем напряжения. Почему? Как же так? Об этом – в будущих статьях. А на сегодня все. Спасибо что читали, удачи, и до новых встреч!

Во всех радиотехнических и электронных устройствах кроме транзисторов и микросхем применяются конденсаторы. В одних схемах их больше, в других меньше, но совсем без конденсаторов не бывает практически ни одной электронной схемы.

При этом конденсаторы могут выполнять в устройствах самые разные задачи. Прежде всего, это емкости в фильтрах выпрямителей и стабилизаторов. С помощью конденсаторов передается сигнал между усилительными каскадами, строятся фильтры низких и высоких частот, задаются временные интервалы в выдержках времени и подбирается частота колебаний в различных генераторах.

Свою родословную конденсаторы ведут от , которую в середине XVIII века в своих опытах использовал голландский ученый Питер ван Мушенбрук. Жил он в городе Лейдене, так что нетрудно догадаться, почему так называлась эта банка.

Собственно это и была обыкновенная стеклянная банка, выложенная внутри и снаружи оловянной фольгой — станиолем. Использовалась она в тех же целях, как и современная алюминиевая, но тогда алюминий открыт еще не был.

Единственным источником электричества в те времена была электрофорная машина, способная развивать напряжение до нескольких сотен киловольт. Вот от нее и заряжали лейденскую банку. В учебниках физики описан случай, когда Мушенбрук разрядил свою банку через цепь из десяти гвардейцев взявшихся за руки.

В то время никто не знал, что последствия могут быть трагическими. Удар получился достаточно чувствительным, но не смертельным. До этого не дошло, ведь емкость лейденской банки была незначительной, импульс получился очень кратковременным, поэтому мощность разряда была невелика.

Как устроен конденсатор

Устройство конденсатора практически ничем не отличается от лейденской банки: все те же две обкладки, разделенные диэлектриком. Именно так на современных электрических схемах изображаются конденсаторы. На рисунке 1 показано схематичное устройство плоского конденсатора и формула для его расчета.

Рисунок 1. Устройство плоского конденсатора

Здесь S — площадь пластин в квадратных метрах, d — расстояние между пластинами в метрах, C — емкость в фарадах, ε — диэлектрическая проницаемость среды. Все величины, входящие в формулу, указаны в системе СИ. Эта формула справедлива для простейшего плоского конденсатора: можно просто расположить рядом две металлические пластины, от которых сделаны выводы. Диэлектриком может служить воздух.

Из этой формулы можно понять, что емкость конденсатора тем больше, чем больше площадь пластин и чем меньше расстояние между ними. Для конденсаторов с другой геометрией формула может быть иной, например, для емкости одиночного проводника или . Но зависимость емкости от площади пластин и расстояния между ними та же, что и у плоского конденсатора: чем больше площадь и чем меньше расстояние, тем больше емкость.

На самом деле пластины не всегда делаются плоскими. У многих конденсаторов, например металлобумажных, обкладки представляют собой алюминиевую фольгу свернутую вместе с бумажным диэлектриком в плотный клубок, по форме металлического корпуса.

Для увеличения электрической прочности тонкая конденсаторная бумага пропитывается изолирующими составами, чаще всего трансформаторным маслом. Такая конструкция позволяет делать конденсаторы с емкостью до нескольких сотен микрофарад. Примерно так же устроены конденсаторы и с другими диэлектриками.

Формула не содержит никаких ограничений на площадь пластин S и расстояние между пластинами d. Если предположить, что пластины можно развести очень далеко, и при этом площадь пластин сделать совсем незначительной, то какая-то емкость, пусть небольшая, все равно останется. Подобное рассуждение говорит о том, что даже просто два проводника, расположенные по соседству, обладают электрической емкостью.

Этим обстоятельством широко пользуются в высокочастотной технике: в некоторых случаях конденсаторы делаются просто в виде дорожек печатного монтажа, а то и просто двух скрученных вместе проводков в полиэтиленовой изоляции. Обычный провод-лапша или кабель также обладают емкостью, причем с увеличением длины она увеличивается.

Кроме емкости C, любой кабель обладает еще и сопротивлением R. Оба этих физических свойства распределены по длине кабеля, и при передаче импульсных сигналов работают как интегрирующая RC — цепочка, показанная на рисунке 2.


Рисунок 2.

На рисунке все просто: вот схема, вот входной сигнал, а вот он же на выходе. Импульс искажается до неузнаваемости, но это сделано специально, для чего и собрана схема. Пока же речь идет о влиянии емкости кабеля на импульсный сигнал. Вместо импульса на другом конце кабеля появится вот такой «колокол», а если импульс короткий, то он может и вовсе не дойти до другого конца кабеля, вовсе пропасть.

Исторический факт

Здесь вполне уместно вспомнить историю о том, как прокладывали трансатлантический кабель. Первая попытка в 1857 году потерпела неудачу: телеграфные точки — тире (прямоугольные импульсы) искажались так, что на другом конце линии длиной 4000 км разобрать ничего не удалось.

Вторая попытка была предпринята в 1865 году. К этому времени английский физик У. Томпсон разработал теорию передачи данных по длинным линиям. В свете этой теории прокладка кабеля оказалась более удачной, сигналы принять удалось.

За этот научный подвиг королева Виктория пожаловала ученого рыцарством и титулом лорда Кельвина. Именно так назывался небольшой город на побережье Ирландии, где начиналась прокладка кабеля. Но это просто к слову, а теперь вернемся к последней букве в формуле, а именно, к диэлектрической проницаемости среды ε.

Немножко о диэлектриках

Эта ε стоит в знаменателе формулы, следовательно, ее увеличение повлечет за собой возрастание емкости. Для большинства используемых диэлектриков, таких как воздух, лавсан, полиэтилен, фторопласт эта константа практически такая же, как у вакуума. Но вместе с тем существует много веществ, диэлектрическая проницаемость которых намного выше. Если воздушный конденсатор залить ацетоном или спиртом, то его емкость возрастет раз в 15…20.

Но подобные вещества обладают кроме высокой ε еще и достаточно высокой проводимостью, поэтому такой конденсатор заряд держать будет плохо, он быстро разрядится сам через себя. Это вредное явление называется током утечки. Поэтому для диэлектриков разрабатываются специальные материалы, которые позволяют при высокой удельной емкости конденсаторов обеспечивать приемлемые токи утечки. Именно этим и объясняется такое разнообразие видов и типов конденсаторов, каждый из которых предназначен для конкретных условий.

Наибольшей удельной емкостью (соотношение емкость / объем) обладают . Емкость «электролитов» достигает до 100 000 мкФ, рабочее напряжение до 600В. Такие конденсаторы работают хорошо только на низких частотах, чаще всего в фильтрах источников питания. Электролитические конденсаторы включаются с соблюдением полярности.

Электродами в таких конденсаторах является тонкая пленка из оксида металлов, поэтому часто эти конденсаторы называют оксидными. Тонкий слой воздуха между такими электродами не очень надежный изолятор, поэтому между оксидными обкладками вводится слой электролита. Чаще всего это концентрированные растворы кислот или щелочей.

На рисунке 3 показан один из таких конденсаторов.

Рисунок 3. Электролитический конденсатор

Чтобы оценить размеры конденсатора рядом с ним сфотографировался простой спичечный коробок. Кроме достаточно большой емкости на рисунке можно разглядеть еще и допуск в процентах: ни много ни мало 70% от номинальной.

В те времена, когда компьютеры были большими и назывались ЭВМ, такие конденсаторы стояли в дисководах (по-современному HDD). Информационная емкость таких накопителей теперь может вызвать лишь улыбку: на двух дисках диаметром 350 мм хранилось 5 мегабайт информации, а само устройство весило 54 кг.

Основным назначением показанных на рисунке суперконденсаторов был вывод магнитных головок из рабочей зоны диска при внезапном отключении электроэнергии. Такие конденсаторы могли хранить заряд несколько лет, что было проверено на практике.

Чуть ниже с электролитическими конденсаторами будет предложено проделать несколько простых опытов, чтобы понять, что может делать конденсатор.

Для работы в цепях переменного тока выпускаются неполярные электролитические конденсаторы, вот только достать их почему-то очень непросто. Чтобы как-то эту проблему обойти, обычные полярные «электролиты» включают встречно-последовательно: плюс-минус-минус-плюс.

Если полярный электролитический конденсатор включить в цепь переменного тока, то сначала он будет греться, а потом раздастся взрыв. Отечественные старые конденсаторы разлетались во все стороны, импортные же имеют специальное приспособление, позволяющее избежать громких выстрелов. Это, как правило, либо крестовая насечка на донышке конденсатора, либо отверстие с резиновой пробкой, расположенное там же.

Очень не любят электролитические конденсаторы повышенного напряжения, даже если полярность соблюдена. Поэтому никогда не надо ставить «электролиты» в цепь, где предвидится напряжение близкое к максимальному для данного конденсатора.

Иногда в некоторых, даже солидных форумах, начинающие задают вопрос: «На схеме означен конденсатор 470µF * 16V, а у меня есть 470µF * 50V, можно ли его поставить?». Да, конечно можно, вот обратная замена недопустима.

Конденсатор может накапливать энергию

Разобраться с этим утверждением поможет простая схема, показанная на рисунке 4.

Рисунок 4. Схема с конденсатором

Главным действующим лицом этой схемы является электролитический конденсатор C достаточно большой емкости, чтобы процессы заряда — разряда протекали медленно, и даже очень наглядно. Это дает возможность наблюдать работу схемы визуально с помощью обычной лампочки от карманного фонаря. Фонари эти давно уступили место современным светодиодным, но лампочки для них продаются до сих пор. Поэтому, собрать схему и провести простые опыты очень даже просто.

Может быть, кто-то скажет: «А зачем? Ведь и так все очевидно, да если еще и описание почитать…». Возразить тут, вроде, нечего, но любая, даже самая простая вещь остается в голове надолго, если ее понимание пришло через руки.

Итак, схема собрана. Как она работает?

В положении переключателя SA, показанном на схеме, конденсатор C заряжается от источника питания GB через резистор R по цепи: +GB __ R __ SA __ C __ -GB. Зарядный ток на схеме показан стрелкой с индексом iз. Процесс заряда конденсатора показан на рисунке 5.

Рисунок 5. Процесс заряда конденсатора

На рисунке видно, что напряжение на конденсаторе возрастает по кривой линии, в математике называемой экспонентой. Ток заряда прямо-таки зеркально отражает напряжение заряда. По мере того, как напряжение на конденсаторе растет, ток заряда становится все меньше. И только в начальный момент соответствует формуле, показанной на рисунке.

Через некоторое время конденсатор зарядится от 0В до напряжения источника питания, в нашей схеме до 4,5В. Весь вопрос в том, как это время определить, сколько ждать, когда же конденсатор зарядится?

Постоянная времени «тау» τ = R*C

В этой формуле просто перемножаются сопротивление и емкость последовательно соединенных резистора и конденсатора. Если, не пренебрегая системой СИ, подставить сопротивление в Омах, емкость в Фарадах, то результат получится в секундах. Именно это время необходимо для того, чтобы конденсатор зарядился до 36,8% напряжения источника питания. Соответственно для заряда практически до 100% потребуется время 5* τ.

Часто, пренебрегая системой СИ, подставляют в формулу сопротивление в Омах, а емкость в микрофарадах, тогда время получится в микросекундах. В нашем случае результат удобнее получить в секундах, для чего придется микросекунды просто умножить на миллион, а проще говоря, переместить запятую на шесть знаков влево.

Для схемы, показанной на рисунке 4, при емкости конденсатора 2000мкФ и сопротивлении резистора 500Ω постоянная времени получится τ = R*C = 500 * 2000 = 1000000 микросекунд или ровно одна секунда. Таким образом, придется подождать приблизительно 5 секунд, пока конденсатор зарядится полностью.

Если по истечении указанного времени переключатель SA перевести в правое положение, то конденсатор C разрядится через лампочку EL. В этот момент получится короткая вспышка, конденсатор разрядится и лампочка погаснет. Направление разряда конденсатора показано стрелкой с индексом iр. Время разряда также определяется постоянной времени τ. График разряда показан на рисунке 6.

Рисунок 6. График разряда конденсатора

Конденсатор не пропускает постоянный ток

Убедиться в этом утверждении поможет еще более простая схема, показанная на рисунке 7.


Рисунок 7. Схема с конденсатором в цепи постоянного тока

Если замкнуть переключатель SA, то последует кратковременная вспышка лампочки, что свидетельствует о том, что конденсатор C зарядился через лампочку. Здесь же показан и график заряда: в момент замыкания переключателя ток максимальный, по мере заряда конденсатора уменьшается, а через некоторое время прекращается совсем.

Если конденсатор хорошего качества, т.е. с малым током утечки (саморазряда) повторное замыкание выключателя к вспышке не приведет. Для получения еще одной вспышки конденсатор придется разрядить.

Конденсатор в фильтрах питания

Конденсатор ставится, как правило, после выпрямителя. Чаще всего выпрямители делаются двухполупериодными. Наиболее распространенные схемы выпрямителей показаны на рисунке 8.

Рисунок 8. Схемы выпрямителей

Однополупериодные выпрямители также применяются достаточно часто, как правило, в тех случаях, когда мощность нагрузки незначительна. Самым ценным качеством таких выпрямителей является простота: всего один диод и обмотка трансформатора.

Для двухполупериодного выпрямителя емкость конденсатора фильтра можно рассчитать по формуле

C = 1000000 * Po / 2*U*f*dU, где C емкость конденсатора мкФ, Po мощность нагрузки Вт, U напряжение на выходе выпрямителя В, f частота переменного напряжения Гц, dU амплитуда пульсаций В.

Большое число в числителе 1000000 переводит емкость конденсатора из системных Фарад в микрофарады. Двойка в знаменателе представляет собой число полупериодов выпрямителя: для однополупериодного на ее месте появится единица

C = 1000000 * Po / U*f*dU,

а для трехфазного выпрямителя формула примет вид C = 1000000 * Po / 3*U*f*dU.

Суперконденсатор — ионистор

В последнее время появился новый класс электролитических конденсаторов, так называемый . По своим свойствам он похож на аккумулятор, правда, с несколькими ограничениями.

Заряд ионистора до номинального напряжения происходит в течение короткого времени, буквально за несколько минут, поэтому его целесообразно использовать в качестве резервного источника питания. По сути ионистор прибор неполярный, единственное, чем определяется его полярность это зарядкой на заводе — изготовителе. Чтобы в дальнейшем эту полярность не перепутать она указывается знаком +.

Большую роль играют условия эксплуатации ионисторов. При температуре 70˚C при напряжении 0,8 от номинального гарантированная долговечность не более 500 часов. Если же прибор будет работать при напряжении 0,6 от номинального, а температура не превысит 40 градусов, то исправная работа возможна в течение 40 000 часов и более.

Наиболее распространенное применение ионистора это источники резервного питания. В основном это микросхемы памяти или электронные часы. В этом случае основным параметром ионистора является малый ток утечки, его саморазряд.

Достаточно перспективным является использование ионисторов совместно с солнечными батареями. Здесь также сказывается некритичность к условию заряда и практически неограниченное число циклов заряд-разряд. Еще одно ценное свойство в том, что ионистор не нуждается в обслуживании.

Пока получилось рассказать, как и где работают электролитические конденсаторы, причем, в основном в цепях постоянного тока. О работе конденсаторов в цепях переменного тока будет рассказано в другой статье — .

Формула емкостного сопротивления конденсатора в цепи переменного тока

Электросопротивление — это параметр в электротехнике, характеризующий возможность вещества препятствовать прохождению электричества.

В зависимости от качеств материала, электросопротивляемость может уменьшаться до крайне маленьких величин (микромилиОмы — у проводников, металлов) или повышаться до огромных значений (ГигаОмы — изоляторов, диэлектриков). Величина противоположная сопротивлению — проводимость.

Что такое

Цепь, по которой протекает непостоянный ток, обладает полным сопротивлением. Вычисляется оно по сумме активного и реактивного сопротивлений, возведенных в квадрат.

Формула вычисления

Графическое изображение этой формулы представляет собой треугольник. Его катеты представлены активным и реактивным сопротивлениями, а гипотенуза полным электросопротивлением.

Графическое отображение формулы

Емкостное электросопротивление (Xc) является одним из видов реактивного сопротивления. Этот показатель характеризует противодействие электроемкости в цепи электротоку с переменными параметрами.

Преобразование электроэнергии в тепловую в момент протекания электричества сквозь емкость не возникает (свойство реактивного сопротивления). Вместо этого осуществляется передача энергии электрического тока электрическому полю и обратно.

Потерь энергии при таком обмене не происходит.

Емкостное сопротивление конденсатора можно сравнить с кастрюлей, наполняемой жидкостью, при полном заполнении ее объема она переворачивается, выливая содержимое, а затем наполняется заново. После достижения максимального заряда конденсатора происходит разрядка, затем он заряжается вновь.

Дополнительная информация: Конденсатор цепи способен накопить лишь ограниченную величину заряда до перемены полярности напряжения.

По данной причине непостоянный ток не падает до нуля, важное отличие от постоянного электричества.

Низкие значения частоты тока соответствуют низким показателям заряда, накопленного конденсатором, низким значениям противодействия электричеству, что придает реактивные свойства.

По сути, Xc — это противостояние электродвижущей силы конденсатора, уровню его заряда.

От чего зависит сопротивление конденсаторов цепей переменного тока

Показатели его, зависят не только от емкостных характеристик последнего, но и от частотной характеристики электротока, протекающего по цепи.

Когда речь идет о сопротивлении резистора, то говорится о параметрах самого резистора, например, материале, форме, но полностью отсутствует взаимосвязь сопротивления его и показателей частоты электричества цепи (речь идет об идеальном резисторе, паразитные параметры которому не характерны).

Когда речь идет об устройстве накопления энергии и заряда электрического поля — все иначе. Конденсатор одной и той же емкости при разных частотах тока обладает неодинаковым уровнем сопротивления. Амплитуда протекающего через него электричества при постоянной амплитуде напряжения обладает разной величиной.

Вычисление Xc

Рассматривая эту формулу сопротивления конденсатора в цепи переменного тока, к каким выводам можно прийти? При повышении частотных показателей сигнала, электросопротивляемость конденсатора снижается.

При повышении емкостных характеристик устройства для накопления заряда и энергии электрического поля Xc переменного электричества, проходящего сквозь него, будет стремиться вниз.

График, отображающий эту величину конденсатора при непостоянном токе цепи, имеет форму гиперболы

Момент приближения значений частоты к нулевым отметкам на оси (когда переменный электроток становится похож своими параметрами на постоянный), сопровождается возрастанием Xc конденсатора до беспредельных величин.

Это действительно так: известно, что конденсатор сети постоянного тока является фактически разрывом цепи. Реальная электросопротивляемость, естественно, не бесконечна, ее ограничивает уровень конденсаторной утечки.

Но величины его остаются на высоком уровне, который невозможно не учитывать.

При возрастании цифр частоты до уровня бесконечных значений, емкостное сопротивление электроконденсатора стремится к нулевым отметкам. Такое характеризует идеальные модели.

В реальных условиях конденсатор имеет неприятные характеристики (такие как индуктивность и сопротивления утечек), поэтому снижение емкостного сопротивления происходит до определенных значений, после которых оно возрастает.

Обратите внимание! При подключении конденсатора к цепочке электричества с переменными параметрами, его мощность не тратится, потому что фазовые характеристики напряжения и силы тока сдвинуты на 90° в отношении друг друга.

В одну четверть периода происходит зарядка электроконденсатора (энергия запасается в его электрополе), в следующее время происходит его разрядка, энергия поступает обратно в цепочку. Его электросопротивляемость является безваттной, реактивной.

Причины ёмкостного сопротивления

Причиной возникновения сопротивления емкостного считается уровень напряжения, возникающий на конденсаторе в процессе его заряда. Вектор его действия встречен вектору напряжения источника электричества, потому создает помеху воспроизведению электротока этим источником.

Как рассчитать Xc

Сила тока цепи с постоянными показателями напряжения в момент работы электроконденсатора равно 0. Ее значения в цепи с переменным напряжением после подключения конденсатора I ? 0. В итоге, цепочке с непостоянным напряжением конденсатор придает Xc меньшее, чем цепочке с неизменным показателем напряжения.

Формула вычисления показателя напряжения за одну секунду

Формула расчета величины силы электротока за мгновение

Получается, что изменения напряжения отличаются по фазе от изменений тока на π/2.

По закону, сформулированному Омом, показатели силы электротока находятся в прямой пропорциональной зависимости от величины напряжения цепи. Формула вычисления наибольших величин напряженности и силы тока:

Наибольшие величины напряженности и силы тока можно рассчитывать по формуле

Окончательная формула расчета емкостного сопротивления в цепи переменного тока

  • ω = 2πf.
  • f — показатель частоты непостоянного тока, измеряется в герцах;
  • ω — показатель угловой частоты тока;
  • С — размер конденсатора в фарадах.

Важно! Xc не выступает параметром проводника, оно находится в зависимости от такой характеристики электроцепи, как частота электротока.

Повышение значений данной величины вызывает рост пропускающей способности конденсатора (предел его сопротивления току непостоянному понижается).

Представим, к цепи подключен конденсатор, емкостью 1 мкФ. Необходимо вычислить, уровень емкостного сопротивления при величине частоты 50 Гц и как изменится емкостное сопротивление цепи переменного тока при частоте 1 кГц. Амплитуда напряжения, подведенного к конденсатору, составляет 50 В.

После введения данных в формулу, определяющую Xc, и получаются значения:

Результат для частоты 50 Гц

Результат для 1 кГц

Емкостное сопротивление приравнивается к соотношению отклонений колебаний напряжения зажимов электрической цепочки с емкостными параметрами (с небольшими индуктивным и активным сопротивлениями) к колебаниям электротока цепочки. Она равнозначна электроконденсатору.

В чем измеряется емкостное электросопротивление

R представлено отношением напряжения к силе тока замкнутой электрической цепи, по закону Ома. Единицы измерения — Ом. Xc, как его разновидность, тоже измеряется в Омах.

Конденсаторы применяются при изготовлении фильтров. При параллельном присоединении к цепи, он способен задерживать высокие частоты, при последовательном удаляет низкие.

Также они используются с целью отсечения переменной части от постоянной. Он незаменим в радиотехнике, при производстве датчиков приближения, для контроля процессов производства.

Технологии, обладающие выше описанными свойствами, используются во всех областях промышленности.

Источник: https://rusenergetics.ru/ustroistvo/yomkostnoe-soprotivlenie

Сопротивление конденсатора

Господа, сегодняшнюю статью можно считать в некотором роде продолжением предыдущей. Сначала я даже хотел поместить весь этот материал в одну статью. Но его получилось довольно много, на горизонте были новые проекты, и я в итоге разделил его на две.

Итак, сегодня мы поговорим про сопротивление конденсатора переменному току.

Мы получим выражение, по которому можно будет рассчитать, чему равно сопротивление любого конденсатора, включенного в цепь с переменным током, а в конце статьи рассмотрим несколько примеров такого расчета.

Сразу оговорюсь про одну важную вещь. Вообще говоря, реальный конденсатор обладает помимо емкостного сопротивления еще резистивным и индуктивным.

На практике все это надо обязательно учитывать, потому что возможны ситуации (обычно связанные с ростом частоты сигнала), когда конденсатор перестает быть конденсатором и превращается… в некое подобие катушки индуктивности . При проектировании схем этот момент обязательно надо иметь в виду. Согласитесь, господа, крайне неприятно поставить в схему конденсатор и потом столкнуться с тем, что из-за высокой частоты он ведет себя и не как конденсатор вовсе, а как самый настоящий дроссель. Это, безусловно, очень важная тема, но сегодня речь пойдет не о ней. В сегодняшней статье мы будем говорить непосредственно про емкостное сопротивление конденсатора. То есть мы будем считать его идеальным, без каких бы то ни было паразитных параметров вроде индуктивности или активного сопротивления.

Давайте представим, что у нас есть конденсатор, который включен в цепь с переменным током. В цепи больше нет никаких компонентов, только один конденсатор и все (рисунок 1).

Рисунок 1 – Конденсатор в цепи переменного тока

К его обкладкам приложено некоторое переменное напряжение U(t), и через него течет некоторый ток I(t). Зная одно, можно без проблем найти другое.

Для этого надо всего лишь вспомнить прошлую статью про конденсатор в цепи переменного тока, там мы про все это довольно подробно говорили.

Будем полагать, что ток через конденсатор изменяется по синусоидальному закону вот так

  • В прошлой статье мы пришли к выводу, что если ток изменятся вот по такому закону, то напряжение на конденсаторе должно меняться следующим образом

Пока что ничего нового мы не записали, это все дословное повторение выкладок из предыдущей статьи. А сейчас самое время их немного преобразовать, придать им чуть другой облик.

Если говорить конкретно, то нужно перейти к комплексному представлению сигналов! Помните, на эту тему была отдельная статья? В ней я говорил, что она нужна для понимания некоторых моментов в дальнейших статьях. Вот как раз и наступил тот момент, когда пора вспомнить все эти хитрые мнимые единицы.

Если говорить конкретно, то сейчас нам потребуется показательная запись комплексного числа. Как мы помним из статьи про комплексные числа в электротехнике, если у нас есть синусоидальный сигнал вида

  1. то его можно представить в показательной форме вот так

Почему это так, откуда взялось, что здесь какая буковка значит – обо всем уже подробно говорили. Для повторения можно перейти по ссылке и еще раз со всем ознакомиться.

Давайте-ка теперь применим это комплексное представление для нашей формулы напряжения на конденсаторе. Получим что-то типа такого

Теперь, господа, я хотел бы вам рассказать еще про один интересный момент, который, наверное, следовало бы описать в статье про комплексные числа в электротехнике. Однако тогда я про него как-то позабыл, поэтому давайте рассмотрим его сейчас.

Давайте представим, что t=0. Это приведет к исключению из расчетов времени и и частоты, и мы переходим к так называемым комплексным амплитудам сигнала. Безусловно, это не значит, что сигнал из переменного становится постоянным.

Нет, он все так же продолжает изменяться по синусу с той же самой частотой. Но бывают моменты, когда частота нам не очень важна, и тогда лучше от нее избавиться и работать только с амплитудой сигнала. Сейчас как раз такой момент.

Поэтому полагаем t=0 и получаем комплексную амплитуду напряжения

  • Давайте раскроем скобки в экспоненте и воспользуемся правилами работы с показательными функциями.

Итак, у нас имеется три множителя. Будем разбираться со всеми по порядку. Объединим первые два и запишем выражение следующего вида

Что мы вообще такое записали? Правильно, комплексную амплитуду тока через конденсатор. Теперь выражение для комплексной амплитуды напряжения принимает вид

Результат, к которому мы стремимся, уже близок, но остается еще один не очень приятный множитель с экспонентой. Как с ним быть? А, оказывается, очень просто. И снова нам на помощь придет статья по комплексным числам в электротехнике, не зря ж я ее писал . Давайте преобразуем этот множитель, воспользовавшись формулой Эйлера:

Да, вся эта хитрая экспонента с комплексными числами в показателе превращается всего лишь в мнимую единичку, перед которой стоит знак минус. Согласен, возможно, осознать это не так просто, но тем не менее математика говорит, что это так. Поэтому результирующая формула у нас принимает вид

  1. Давайте выразим из этой формулы ток и приведем выражение к виду, соответствующему закону Ома. Получим

Как мы помним из статьи про закон Ома, у нас ток равнялся напряжению, деленному на сопротивление.

Так вот, здесь практически то же самое! Ну, за исключением того, что у нас ток и напряжение – переменные и представлены через комплексные амплитуды. Кроме того, не забываем, что ток течет у нас через конденсатор.

Поэтому, выражение, которое стоит в знаменателе, можно рассматривать как емкостное сопротивление конденсатора переменному току:

Да, выражение для сопротивления конденсатора имеет вот такой вот вид. Оно, как вы можете заметить, комплексное. Об этом свидетельствует буковка j в знаменателе дроби.

А что значит эта комплексность? На что она влияет и что показывает? А показывает она, господа, исключительно сдвиг фаз в 90 градусов между током и напряжением на конденсаторе. А именно, ток на 90 градусов опережает напряжение. Этот вывод не является для нас новостью, про все это было подробно рассказано в прошлой статье.

Чтобы это лучше осознать, надо теперь мысленно пройтись от полученной формулы вверх к тому моменту, где у нас это j возникло. В процессе подъема вы увидите, что мнимая единица j возникло из формулы Эйлера из-за того, что там был компонент . Формула Эйлера у нас возникла из комплексного представления синусоиды.

А в исходной синусоиде как раз был заложен сдвиг фазы в 90 градусов тока относительно напряжения. Как-то так. Вроде все логично и ничего лишнего не возникло.

Теперь может возникнуть два совершенно логичных вопроса: как работать с таким представлением и в чем его выгода? Да и вообще, пока лишь какие-то дико абстрактные буковки и нифига не ясно, как взять и оценить сопротивление какого-нибудь конкретно конденсатора, который мы купили в магазине и воткнули в схему. Давайте разбираться постепенно.

Как мы уже говорили, буковка j в знаменателе говорит нам лишь о сдвиге фаз тока и напряжения. Но она не влияет на амплитуды тока и напряжения. Соответственно, если сдвиг фаз нас не интересует, то можно исключить эту буковку из рассмотрения и получить более простое выражение абсолютно без всяких комплексностей:

Согласитесь, жить стало чуточку легче. Это выражение позволяет рассчитать сопротивление конденсатора для конкретной емкость и частоты сигнала. Заметьте, господа, интересный факт. Сопротивление конденсатора, оказывается, зависит не только от самого конденсатора (а именно его емкости), но и от частоты протекающего тока.

Если вспомнить обычные резисторы, то в них у нас сопротивление зависело только от самого резистора, материала, формы и всего такого прочего, но не зависело от частоты (разумеется, мы говорим сейчас про идеальные резисторы, без всяких паразитных параметров). Здесь все по-другому.

Один и тот же конденсатор на разной частоте будет иметь разное сопротивление и через него будет течь ток разной амплитуды при одной и той же амплитуде напряжения.

Что еще мы можем сказать, глядя на эту формулу? Например, то, что чем больше частота сигнала, тем меньше для него сопротивление конденсатора. И чем больше емкость конденсатора, тем меньше его сопротивление переменному току.

По аналогии с резисторами, сопротивление конденсаторов измеряется все так же в Омах. Однако всегда следует помнить, что это немного другое сопротивление, его называют реактивным. И другое оно в первую очередь из-за того самого пресловутого j в знаменателе, то есть из-за сдвига фазы.

У «обычных» (которые называют активными) Омов такого сдвига нет, там напряжение четко совпадает по фазе с током. Давайте построим график зависимости сопротивления конденсатора от частоты. Для определенности емкость конденсатора возьмем фиксированной, скажем, 1 мкФ. График представлен на рисунке 2.

  • Рисунок 2 (кликабельно) – Зависимость сопротивления конденсатора от частоты
  • На рисунке 2 мы видим, что сопротивление конденсатора переменному току убывает по закону гиперболы.

При стремлении частоты к нулю (то есть фактически при стремлении переменного току к постоянному) сопротивление конденсатора стремится к бесконечности.

Это и логично: мы все помним, что для постоянного тока конденсатор фактически представляет собой разрыв цепи. На практике оно, конечно, не бесконечно, а ограничено сопротивлением утечки конденсатора.

Тем не менее, оно все равно очень велико и часто его и считают бесконечно большим.

При стремлении частоты к бесконечности, сопротивление конденсатора стремится к нулю. Это все в теории, конечно.

На практике реальный конденсатор обладает рядом паразитных параметров (в частности, паразитная индуктивности и сопротивление утечки), из-за чего сопротивление уменьшается только лишь до некоторой определенной частоты, а потом начинает наоборот расти. Но об этом более подробно в другой раз.

Есть еще один вопрос, который хотелось бы обговорить, прежде чем начинать рассмотрение примеров. Зачем вообще писать букву j в знаменателе сопротивления? Не достаточно ли просто всегда помнить про сдвиг фаз, а в записи использовать числа без этой мнимой единицы? Оказывается, нет.

Представим себе цепь, где одновременно присутствуют резистор и конденсатор. Скажем, они соединены последовательно. И вот тут-то как раз мнимая единичка рядом с емкостью не позволит просто так взять и сложить активное и реактивное сопротивление в одно действительное число.

Общее сопротивление такой цепочки будет комплексным, причем состоящим как из действительной части, так и из мнимой. Действительная часть будет обусловлена резистором (активными сопротивлением), а мнимая – емкостью (реактивным сопротивлением).

Впрочем, это все тема для другой статьи, сейчас не будем в это углубляться. Давайте лучше перейдем к примерам.

Пусть у нас есть конденсатор емкостью, скажем C=1 мкФ. Требуется определить его сопротивление на частоте f1=50 Гц и на частоте f2=1 кГц. Кроме того, следует определить амплитуду тока с учетом того, что амплитуда приложенного к конденсатору напряжения равна Um=50 В. Ну и построить графики напряжения и тока.

  1. Собственно, задачка эта элементарная. Подставляем циферки в формулу для сопротивления и получаем для частоты f1=50 Гц сопротивление, равное
  2. А для частоты f2=1 кГц сопротивление будет
  3. По закону Ома находим величину амплитуды тока для частоты f1=50 Гц
  4. Аналогично для второй частоты f2=1 кГц
  5. Теперь мы легко можем записать законы изменения тока и напряжения, а также построить графики для этих двух случаев. Полагаем, что напряжение у нас изменяется по закону синуса для первой частоты f1=50 Гц следующим образом
  6. А для второй частоты f2=1 кГц вот так
  7. Дальше мы помним, что ток в конденсаторе опережает напряжение на . Поэтому с учетом этого можем записать закон изменения тока через конденсаторы для первой частоты f1=50 Гц
  8. и для частоты f2=1 кГц
  9. Графики тока и напряжения для частоты f1=50 Гц представлены на рисунке 3
  10. Рисунок 3 (кликабельно) – Напряжение на конденсаторе и ток через конденсаторе, f1=50 Гц
  11. Графики тока и напряжения для частоты f2=1 кГц представлены на рисунке 4
  12. Рисунок 4 (кликабельно) – Напряжение на конденсаторе и ток через конденсаторе, f2=1 кГц

Итак, господа, мы сегодня познакомились с таким понятием, как сопротивление конденсатора переменному току, научились его считать и закрепили полученные знания парочкой примеров. На сегодня все. Спасибо что прочитали, всем огромной удачи и пока!

Источник: http://myelectronix.ru/peremennyy-tok/63-soprotivlenie-kondensatora

Емкостное сопротивление конденсатора формула расчёта и последовательность соединения в цепи

Емкостное сопротивление конденсатора – величина, измеряемая в омах, создается непосредственно самим конденсатором, который включен в любую цепь. Оно должно иметь большую величину, то есть быть большим.

Если на них происходит подача переменного тока, в устройстве происходят процессы заряда и последующего разряда. Последнее происходит по требованию цепи. При включении электрического тока, напряжение будет равно 0.

Само устройство при этом начнет заряжаться, следовательно его величина напряжения постепенно растет. В случае необходимости, при достижении максимального заряда, произойдет разряд конденсатора.

В статье, посвященной теме расчета сопротивления конденсатора, приведена вся информация о процессе, как происходит заряд-разряд. В качестве бонуса есть интересный материал по теме, который можно скачать, и видеоролик в конце статьи.

Формула сопротивления конденсаторов.

Формула сопротивления

Формула ёмкостного сопротивления выводится следующим образом:

  • Вначале следует вычислить угловую частоту. Для этого частоту протекающего по цепи тока (в герцах) необходимо умножить на удвоенное число «пи».
  • Затем полученное число следует перемножить на ёмкость конденсатора в фарадах.

Чтобы получить значение ёмкостного сопротивления в омах, следует разделить единицу на число, полученное после умножения угловой частоты на ёмкость.

Из этой формулы вытекает, что чем больше ёмкость конденсатора или частота переменного тока, тем меньше его сопротивление. Когда частота будет равна нулю (постоянный ток), ёмкостное сопротивление станет бесконечно большим.

Конденсатор очень большой ёмкости будет проводить ток в широком диапазоне частот.

Применение на практике

Свойства конденсатора используются при конструировании различных фильтров. Действие ёмкостного сопротивления в этом случае зависит от способа подключения детали:

  • Если он присоединён параллельно нагрузке, то получится фильтр, задерживающий высокие частоты. С их ростом падает сопротивление конденсатора. Соответственно, нагрузка на высоких частотах шунтируется сильнее, чем на низких.
  • Если деталь подключена последовательно с нагрузкой, то получится фильтр, задерживающий низкие частоты. Эта схема также не пропускает постоянное напряжение.
  • Ещё одна область применения — отделение переменной составляющей от постоянной. Например, в оконечных каскадах усилителей звуковой частоты. Чем выше ёмкость, тем более низкую частоту способен воспроизвести подключённый громкоговоритель.

В фильтрах электропитания, наряду с ёмкостным сопротивлением, используется также свойство накопления и отдачи заряда. В момент повышения нагрузки заряженная ёмкость фильтра разряжается, отдавая дополнительную энергию.

Она также осуществляет подавление пульсаций и прочих паразитных сигналов, пропуская их через себя и замыкая на общий провод.

Таким образом, обеспечивается сглаживание и поддержание напряжения на нагрузке в заданных пределах, и устранение нежелательных междукаскадных связей, вызывающих нестабильную работу.

Измерение сопротивления конденсаторов.

Характеристики прибора

Важнейшей характеристикой накопительного прибора является ёмкость. От неё зависит время заряда при подключении устройства к источнику тока. Время разряда напрямую связано со значением сопротивления нагрузки: чем оно выше, тем быстрее происходит процесс отдачи накопленной энергии. Определяется эта ёмкость следующим выражением:

Будет интересно➡  Конденсатор — простыми словами о сложном

C = E*Eo*S / d, где E — относительная диэлектрическая проницаемость среды (справочная величина), S — площадь пластин, d — расстояние между ними. Кроме ёмкости конденсатор характеризуется рядом параметров, такими как:

  • удельная ёмкость — определяет отношение величины ёмкости к массе диэлектрика;
  • рабочее напряжение — номинальное значение, которое может выдержать устройство при подаче его на обкладки элемента;
  • температурная стабильность — интервал, в котором ёмкость конденсатора практически не изменяется;
  • сопротивление изоляции — характеризуется саморазрядом устройства и определяется током утечки;
  • эквивалентное сопротивление — состоит из потерь, образуемых на выводах прибора и слое диэлектрика;
  • абсорбция — процесс возникновения разности потенциалов на обкладках после разряда устройства до нуля;
  • ёмкостное сопротивление — уменьшение проводимости при подаче переменного тока;
  • полярность — из-за физических свойств материала, используемого при изготовлении, конденсатор сможет правильно работать, только если к обкладкам приложен потенциал с определённым знаком;
  • эквивалентная индуктивность — паразитный параметр, появляющийся на контактах устройства и превращающий конденсатор в колебательный контур.

Таблицы максимальных значений емкости конденсаторов.

Импеданс элемента

Общее сопротивление конденсатора (импеданс) переменному сигналу складывается из трёх составляющих: ёмкостного, резистивного и индуктивного сопротивления. Все эти величины при конструировании схем, содержащих накопительный элемент, необходимо учитывать. В ином случае в электрической цепи, при соответствующей обвязке, конденсатор может вести себя как дроссель и находится в резонансе.

Полное сопротивление элемента выражается в формуле Z = (R2 + (Xl-Xc) 2 ) ½, где

  • Xl — индуктивность;
  • Xс — ёмкость;
  • R — активная составляющая.

Последняя возникает из-за появления электродвижущей силы (ЭДС) самоиндукции. Непостоянство тока приводит к изменению магнитного потока, поддерживающего ток ЭДС самоиндукции постоянным.

Это значение определяется индуктивностью L и частотой протекающих зарядов W. Xl = wL = 2*p*f*L. Xc — ёмкостное сопротивление, зависящее от ёмкости накопителя C и частоты тока f.

Xc = 1/wC = ½*p*f*C, где w — круговая частота.

Материал в тему: все о переменном конденсаторе.

Разница между ёмкостным и индуктивным значениями называется реактивным сопротивлением конденсатора: X = Xl-Xc. По формулам можно увидеть, что при увеличении частоты f сигнала начинает преобладать индуктивное значение, при уменьшении — ёмкостное. Поэтому если:

  • X > 0, в элементе проявляются индуктивные свойства;
  • X = 0, в ёмкости присутствует только активная величина;
  • X

Активное сопротивление R связывается с потерями мощности, превращением её электрической энергии в тепловую. Реактивное – с обменом энергии между переменным током и электромагнитным полем. Таким образом, полное сопротивление можно найти, используя формулу Z = R +j*X, где j — мнимая единица.

Пример расчёта

Ёмкостное и индуктивное сопротивления относятся к реактивным, то есть таким, которые не потребляют мощности. Поэтому закон Ома для участка схемы с ёмкостью имеет вид I = U/Xc, где ток и напряжение обозначают действующие значения.

Именно из-за этого конденсаторы используются в цепях для разделения не только постоянных и переменных токов, но и низкой и высокой частот. При этом чем ёмкость будет ниже, тем более высокой частоты сможет пройти ток.

Если же последовательно с конденсатором включено активное сопротивление, то общий импеданс цепи находится как Z = (R 2 +Xc 2 ) ½.

Практическое применение формул можно рассмотреть при решении задачи. Пусть имеется RC цепочка, состоящая из ёмкости C = 1 мкФ и сопротивления R = 5 кОм. Необходимо найти импеданс этого участка и ток цепи, если частота сигнала равна f = 50 Гц, а амплитуда U = 50 В.

Стоит почитать: все об электролитических конденсаторах.

  • В первую очередь понадобится определить сопротивление конденсатора в цепи переменного тока для заданной частоты. Подставив данные в формулу, получим, что для частоты 50 Гц сопротивление будет
  • Xc = 1/ (2*p*F*C) = 1/ (2*3,14*50*1* 10 −6 ) = 3,2 кОм.
  • По закону Ома можно найти ток: I = U /Xc = 50 /3200 = 15,7 мА.

Напряжение берётся изменяемым по закону синуса, поэтому: U (t) = U * sin (2*p*f*t) = 50*sin (314*t). Соответственно, ток будет I (t) = 15,7* 10 −3 + sin (314*t+p/2). Используя полученные результаты, можно построить график тока и напряжения при этой частоте. Общее сопротивление участка цепи находим как Z = (50002+32002)½ = 5 936 Ом =5,9 кОм.

Свойства ёмкостей

Основное свойство состоит в их способности накапливать и отдавать электрический заряд. Оба этих процесса происходят не мгновенно, а за вполне определённый период, который поддаётся расчету. Данное свойство используется для создания различных времязадающих RC цепей.

Если зарядить конденсатор до некоторого значения, то время его разряда через резистор R будет зависеть от ёмкости C. RC цепь Ещё одно распространённое свойство конденсаторов – это возможность ограничивать переменный ток. Вызвана она реактивом этих элементов.

Ёмкость, включенная в цепь переменного тока, ограничивает его до значения I = 2pfCU.

Здесь U – напряжение источника питания. Дополнительная информация. Ёмкость, подключенная параллельно с катушкой, имеющей индуктивный характер сопротивления, называется колебательным контуром. Данная цепь обладает высокой амплитудой колебаний на резонансной частоте. Она применяется для выделения из множества окружающих радиосигналов именно того, на который требуется настроить приём.

Резистор обладает активным (омическим) сопротивлением. Катушка индуктивности и конденсатор обладают реактивным сопротивлением. В цепи переменного тока на конденсаторе ток опережает напряжение на 90 градусов, а на катушке ток отстает от напряжения на 90 градусов. Сопротивление катушки вычисляется по формуле. Сопротивление конденсатора вычисляется по формуле:

В цепи переменного тока на идеальном реактивном сопротивлении не выделяется мощность.

Будет интересно➡  Несколько фактов об электролитических конденсаторах

Z = R + i X , где Z – импеданс, R – величина активного сопротивления , X – величина реактивного сопротивления, i – мнимая единица . В зависимости от величины X какого-либо элемента электрической цепи, говорят о трёх случаях:

  • X > 0 – элемент проявляет свойства индуктивности .
  • X = 0 – элемент имеет чисто активное сопротивление .

Величина реактивного сопротивления может быть выражена через величины индуктивного и ёмкостного сопротивлений.

Индуктивное сопротивление (X L ) обусловлено возникновением ЭДС самоиндукции . Электрический ток создает магнитное поле.

Изменение тока, и как следствие изменение магнитного поля, вызывает ЭДС самоиндукции, которая препятствует изменению тока. Величина индуктивного сопротивления зависит от индуктивности элемента и частоты протекающего тока.

Ёмкостное сопротивление (X C ). Величина ёмкостного сопротивления зависит от ёмкости элемента С и также частоты протекающего тока.

Заключение

В данной статье были рассмотрены основные вопросы расчета сопротивления конденсаторов.  Больше информации можно найти в скачиваемой версии учебника по электромеханике “Что такое конденсаторы”

В нашей группе ВК можно задавать вопросы и получать на них подробные ответы от профессиональных электронщиков. Чтобы подписаться на группу, вам необходимо будет перейти по следующей ссылке: https://vk.com/electroinfonet. В завершение статьи хочу выразить благодарность источникам, откуда мы черпали информацию:

www.amperof.ru

www.eduspb.com

www.beasthackerz.ru

www.electroandi.ru

www.websor.ru

Источник: https://ElectroInfo.net/kondensatory/formula-raschjota-soprotivlenija-kondensatora.html

Формула емкостного сопротивления

После замыкания электрической цепи начинается зарядка, после чего конденсатор сразу же становится источником тока и напряжения, в нем возникает электродвижущая сила – ЭДС. Одно из основных свойств конденсатора очень точно отражает формула емкостного сопротивления.

Данное явление возникает в результате противодействия ЭДС, направленного против источника тока, используемого для зарядки. Источник тока может преодолеть емкостное сопротивление лишь путем существенных затрат его собственной энергии, которая становится энергией электрического поля конденсатора.

При разрядке конденсатор вся энергия возвращается обратно в цепь, превращаясь в энергию электрического тока.

Емкостное сопротивление конденсатора

Конденсаторы относятся к наиболее распространенным элементам, используемым в различных электронных схемах.

Они разделяются на типы, обладающие характерными особенностями, параметрами и индивидуальными свойствами. Простейший конденсатор состоит из двух металлических пластин – электродов, разделенных слоем диэлектрика.

На каждом из них имеется собственный вывод, через который осуществляется подключение к электрической цепи.

Совершенно по-другому на конденсатор воздействует переменный ток, вполне свободно протекающий через емкость. Подобное состояние объясняется постоянными процессами зарядки-разрядки элемента. В этом случае действует не только активное сопротивление проводников, но и емкостное сопротивление самого конденсатора, возникающее как раз в результате его постоянной зарядки и разрядки.

Электрические параметры и свойства конденсаторов могут отличаться, в зависимости от различных факторов. В первую очередь они зависят от размеров и формы изделия, а также от типа диэлектрика.

В разных типах устройств диэлектриком может служить бумага, воздух, пластик, стекло, слюда, керамика и другие материалы.

В электролитических конденсаторах используются алюминий-электролит и тантал-электролит, что обеспечивает им повышенную емкость.

Изменение магнитного потока

Названия других элементов определяются материалами обычных диэлектриков. Поэтому они относятся к категории бумажных, керамических, стеклянных и т.д. Каждый из них, в соответствии с характеристиками и особенностями, применяется в конкретных электронных схемах, с разными параметрами электротока.

В связи с этим, применение керамических конденсаторов необходимо в тех цепях, где требуется фильтрация высокочастотных помех. Электролитические устройства, наоборот, фильтруют помехи при низких частотах.

Если же соединить параллельно оба типа конденсаторов, получится универсальный фильтр, широко применяемый во всех схемах. Несмотря на то, что их емкость является фиксированной величиной, существуют устройства с переменной емкостью, которая достигается путем регулировок за счет изменение взаимного перекрытия пластин.

Типичным примером служат конденсаторы для подстройки, используемые при регулировке радиоэлектронной аппаратуры.

Емкостное сопротивление в цепи переменного тока

При включении конденсатора в цепь постоянного тока, на протяжении короткого периода времени будет наблюдаться течение по цепи зарядного тока. По окончании зарядки, когда напряжение конденсатора будет соответствовать напряжению источника тока, кратковременное течение тока в цепи прекратится.

Таким образом, полностью заряженный конденсатор при постоянном токе будет своеобразным разрывом цепи или сопротивлением с бесконечно большим значением. При переменном токе конденсатор будет вести себя совершенно иначе. Его зарядка в такой цепи будет осуществляться поочередно в разных направлениях.

Течение переменного тока в цепи в это время не прерывается.

Более подробное рассмотрение этого процесса указывает на нулевое значение напряжения в конденсаторе в момент его включения. После поступления к нему переменного напряжения сети начнется зарядка.

В это время сетевое напряжение будет возрастать на протяжении первой четверти периода. По мере того как на обкладках накапливаются заряды, происходит увеличение напряжения самого конденсатора.

После того как сетевое напряжение в конце первой четверти периода станет максимальным, зарядка прекращается и значение тока в цепи станет равным нулю.

Существует формула для определения тока в цепи конденсатора: I = ∆q/∆t, где q является количеством электричества, протекающим по цепи в течение промежутка времени t.

В соответствии с законами электростатики, количество электричества в устройстве составит: q = C x Uc = C x U. В этой формуле С будет емкостью конденсатора, U – напряжением сети, Uc – напряжением на обкладках элемента.

В окончательном виде формула тока в цепи будет выглядеть следующим образом: i = C x (∆Uc/∆t) = C x (∆U/∆t).

При наступлении второй четверти периода произойдет уменьшение сетевого напряжения и начнется разрядка конденсатора. Ток в цепи изменит свое направление и будет течь в обратную сторону.

В следующей половине периода направление сетевого напряжения изменится, наступит перезарядка элемента, а потом он вновь начнет разряжаться.

Ток, присутствующий в цепи с конденсаторной емкостью, будет опережать по фазе напряжение на обкладках на 90 градусов.

Установлено что изменения тока конденсатора происходят со скоростью, находящейся в пропорциональной зависимости с угловой частотой ω. Поэтому в соответствии с уже известной формулой тока в цепи i = C x (∆U/∆t), аналогично получается, что действующее значение тока также будет представлять собой пропорцию между скоростью изменения напряжения и угловой частотой ω: I = 2π x f x C x U.

Далее уже совсем несложно установить значение емкостного сопротивления или реактивного сопротивления емкости: xc = 1/2π x f x C = 1/ ω x C.

Данный параметр вычисляется, когда конденсаторная емкость включается в цепь переменного тока.

Поэтому в соответствии с законом Ома в цепи переменного тока с включенным конденсатором, значение силы тока будет следующим: I = U/xc, а напряжение на обкладках составит: Uc = Ic x xc.

Часть сетевого напряжения, приходящаяся на конденсатор, получила название емкостного падения напряжения. Она известна также, как реактивная слагающая напряжения, обозначаемая символом Uc. Величина емкостного сопротивления хс, так же, как и значение индуктивного сопротивления xi напрямую связана с частотой переменного тока.

Режим короткого замыкания

Источник: https://electric-220.ru/news/formula_emkostnogo_soprotivlenija/2017-05-04-1255

Конденсаторы и цепи переменного тока . Искусство схемотехники. Том 1 [Изд.4-е]

Коль скоро мы начинаем рассматривать изменяющиеся сигналы напряжения и тока, нам необходимо познакомиться с двумя очень занятными элементами, которые не находят применения в цепях постоянного тока, — речь идет о конденсаторах и индуктивностях. Скоро вы убедитесь, что эти компоненты вместе с резисторами являются основными элементами пассивных линейных цепей, составляющих основу почти всей схемотехники.

Особенно следует подчеркнуть роль конденсаторов — без них не обходится почти ни одна схема. Они используются при генерации колебаний, в схемах фильтров, для блокировки и шунтирования сигналов. Их используют в интегрирующих и дифференцирующих схемах. На основе конденсаторов и индуктивностей строят схемы формирующих фильтров для выделения нужных сигналов из фона.

Некоторые примеры подобных схем вы найдете в этой главе, а еще большее число интересных примеров использования конденсаторов и индуктивностей встретится вам в последующих главах.

Приступим к более детальному изучению конденсаторов. Явления, протекающие в конденсаторе, описываются математическими зависимостями, поэтому читателям, которые имеют недостаточную подготовку в области математики, полезно прочитать приложение Б. Не огорчайтесь, если некоторые детали не будут сразу вполне понятны, главное — это общее понимание вопроса.

1.12. Конденсаторы

Конденсатор (рис. 1.27) — это устройство, имеющее два вывода и обладающее следующим свойством:

Q = CU.

Рис. 1.27. Конденсатор.

Конденсатор, имеющий емкость С фарад, к которому приложено напряжение U вольт, накапливает заряд Q кулон на одной пластине и —Q — на другой.

В первом приближении конденсаторы — это частотно-зависимые резисторы. Они позволяют создавать, например, частотно-зависимые делители напряжения. Для решения некоторых задач (шунтирование, связывание контуров) больших знаний о конденсаторе и не требуется, другие задачи (построение фильтров, резонансных схем, накопление энергии) требуют более глубоких знаний. Например, конденсаторы не рассеивают энергию, хотя через них и протекает ток, — дело в том, что ток и напряжение на конденсаторе смещены друг относительно друга по фазе на 90°.

Продифференцировав выражение для Q (см. приложение Б), получим

I = C(dU/dt).

Итак, конденсатор — это более сложный элемент, чем резистор; ток пропорционален не просто напряжению: а скорости изменения напряжения. Если напряжение на конденсаторе, имеющем емкость 1 Ф, изменится на 1 В за 1 с, то получим ток 1 А. И наоборот, протекание тока 1 А через конденсатор емкостью 1 Φ вызывает изменение напряжения на 1 В за 1 с.

Емкость, равная одной фараде, очень велика, и поэтому чаще имеют дело с микрофарадами (мкФ) или пикофарадами (пФ). Для того чтобы сбить с толку непосвященных, на принципиальных схемах иногда опускают обозначения единиц измерения. Их приходится угадывать из контекста. Например, если подать ток 1 мА на конденсатор емкостью 1 мкФ, то напряжение за 1 с возрастет на 1000 В.

Импульс тока продолжительностью 10 мс вызовет увеличение напряжения на конденсаторе на 10 В (рис. 1.28).

Рис. 1.28. Напряжение на конденсаторе изменяется, когда через него протекает ток.

Промышленность выпускает конденсаторы разнообразных форм и размеров, через некоторое время вы познакомитесь с наиболее распространенными представителями этого обширного семейства.

Простейший конденсатор состоит из двух проводников, расположенных на небольшом расстоянии друг от друга (но не соприкасающихся между собой), настоящие простейшие конденсаторы имеют именно такую конструкцию. Чтобы получить большую емкость, нужны большая площадь и меньший зазор между проводниками, обычно для этого один из проводников покрывают тонким слоем изолирующего материала (называемого диэлектриком), для таких конденсаторов используют, например, алитированную (покрытую алюминием) майларовую пленку.

Широкое распространение получили следующие типы конденсаторов: керамические, электролитические (изготовленные из металлической фольги с оксидной пленкой в качестве изолятора), слюдяные (изготовленные из металлизированной слюды). Каждому типу конденсаторов присущи свои качества, краткий перечень отличительных особенностей каждого типа конденсаторов приведен мелким шрифтом в разделе «Конденсаторы». В общем можно сказать, что для некритичных схем подходят керамические и майларовые конденсаторы, в схемах, где требуется большая емкость, применяются танталовые конденсаторы, а для фильтрации в источниках питания используют электролитические конденсаторы.

Параллельное и последовательное соединение конденсаторов. Емкость нескольких параллельно соединенных конденсаторов равна сумме их емкостей. Нетрудно в этом убедиться: приложим напряжение к параллельному соединению, тогда

CU = Q = Q1 + Q2 + Q3 +… =

= C1U + C2U + C3U +…=

= (C1 + C+ C3 +…)·U =

или

С = С1 + С2 + С3 +….

Для последовательного соединения конденсаторов имеем такое же выражение, как для параллельного соединения резисторов:

В частном случае для двух конденсаторов:

С = С1С2/(С1 + С2).

Ток, заряжающий конденсатор (IC·dU/dt), обладает некоторыми особыми свойствами. В отличие от тока, протекающего через резистор, он пропорционален не напряжению, а скорости изменения напряжения (т. е. его производной по времени). Далее, мощность (U умноженное на I), которая связана с протекающим через конденсатор током, не обращается в тепло, а сохраняется в виде энергии внутреннего электрического поля в конденсаторе. При разряде конденсатора происходит извлечение энергии. Эти занятные свойства мы рассмотрим с другой точки зрения, когда будем изучать реактивность (начиная с разд. 1.18).

* * *

КОНДЕНСАТОРЫ

Промышленностью выпускается много типов конденсаторов. Здесь перечислены основные преимущества и недостатки различных типов. Очевидно, что данная оценка имеет несколько субъективный характер (см. таблицу).

Упражнение 1.12. Получите выражение для емкости двух последовательно соединенных конденсаторов.

Подсказка: так как точка соединения конденсаторов не имеет внешних подключений, то заряд, накопленный двумя конденсаторами, должен быть одинаков.

* * *

1.13. RС-цепи: изменения во времени напряжения и тока

Для анализа цепей переменного тока (или в общем случае схем, работающих с изменяющимися напряжениями и токами) можно использовать характеристики двух типов. Во-первых, можно рассматривать изменения напряжения U и тока I во времени, а во-вторых, — изменение амплитуды при изменении частоты сигнала. И те, и другие характеристики имеют свои преимущества, и в каждом практическом случае приходится выбирать наиболее подходящие. Мы начнем изучение цепей переменного тока с временных зависимостей, а в разд. 1.18 перейдем к частотным характеристикам.

Каковы же свойства схем, в состав которых входят конденсаторы? Для того чтобы ответить на этот вопрос, рассмотрим простейшую -цепь (рис. 1.29).

Рис. 1.29.

Воспользуемся полученным ранее выражением для емкости:

C(dU/dt) = I = — U/R.

Это выражение представляет собой дифференциальное уравнение, решение которого имеет вид

U = A·e-t/RC.

Отсюда следует, что если заряженный конденсатор подключить к резистору, то он будет разряжаться так, как показано на рис. 1.30.

Рис. 1.30. Сигнал разряда RС-цепи.

Постоянная времени. Произведение RC называют постоянной времени цепи. Если R измерять в омах, а С — в фарадах, то произведение RC будет измеряться в секундах. Для конденсатора емкостью 1 мкФ, подключенного к резистору сопротивлением 1 кОм, постоянная времени составляет 1 мс, если конденсатор был предварительно заряжен и напряжение на нем составляет 1 В, то при подключении резистора в цепи появится ток, равный 1 мА.

На рис. 1.31 показана несколько иная схема.

Рис. 1.31.

В момент времени t = 0 схема подключается к батарее. Уравнение, описывающее работу такой схемы, выглядит следующим образом:

I = C(dU/dt) = (Uвх U)/R

и имеет решение

UUвх + Ae-t/RC.

Не пугайтесь, если не поняли, как выполнено математическое преобразование. Важно запомнить полученный результат. В дальнейшем мы будем многократно его использовать, не прибегая к математическим выкладкам. Постоянная величина А определяется из начальных условий (рис. 1.32): U = 0 при I = 0, откуда А = — Uвх и UUвх(1 — e-t/RC).

Рис. 1.32.

Установление равновесия. При условии t >> RC напряжение достигает значения Uвх. (Советуем запомнить хорошее практическое правило, называемое правилом пяти RC. Оно гласит: за время, равное пяти постоянным времени, конденсатор заряжается или разряжается на 99 %.) Если затем изменить входное напряжение Uвх (сделать его равным, например, нулю), то напряжение на конденсаторе U будет убывать, стремясь к новому значению по экспоненциальному закону e-t/RC.

Например, если на вход подать прямоугольный сигнал Uвх, то сигнал на выходе U будет иметь форму, показанную на рис. 1.33.

Рис. 1.33. Напряжение, снимаемое с конденсатора (верхние сигналы), при условии, что на него через резистор подается прямоугольный сигнал.

Упражнение 1.13. Докажите, что время нарастания сигнала (время, в течение которого сигнал изменяется от 10 до 90 % своего максимального значения) составляет 2,2RC.

У вас, наверное, возник вопрос: каков закон изменения для произвольного Uвх(t)? Для того чтобы ответить на него, нужно решить неоднородное дифференциальное уравнение (стандартные методы решения таких уравнений здесь не рассматриваются). В результате получим

Согласно полученному выражению, RC— цепь усредняет входное напряжение с коэффициентом пропорциональности e-Δt/RC, где Δt = τ t. На практике, однако, такой вопрос возникает редко. Чаще всего рассматриваются частотные характеристики и определяют, какие изменения претерпевает каждая частотная составляющая входного сигнала. Скоро (разд. 1.18) мы также перейдем к этому немаловажную вопросу. А пока рассмотрим несколько интересных схем, для анализа которых достаточно временных зависимостей.

Упрощение с помощью эквивалентного преобразования Тевенина. Можно было бы приступить к анализу более сложных схем, пользуясь, как и раньше, методом решения дифференциальных уравнений. Однако чаще всего не стоит прибегать к решению дифференциальных уравнений.

Большинство схем можно свести к RC-схеме, показанной на рис. 1.34.

Рис. 1.34.

Пользуясь эквивалентным преобразованием для делителя напряжения, образованного резисторами R1 и R2, можно определить U(t) для скачка входного напряжения Uвх.

Упражнение 1.14. Для схемы, показанной на рис. 1.34, R1R2 = 10 кОм и С = 0,1 мкФ. Определите U(t) и изобразите полученную зависимость в виде графика.

Пример: схема задержки. Мы уже упоминали логические уровни — напряжения, определяющие работу цифровых схем. На рис. 1.35 показано, как с помощью конденсаторов можно получить задержанный импульс.

Рис 1.35. Использование RC-цепи для формирования задержанного цифрового сигнала.

В виде треугольников изображены КМОП-буферные усилители. Они дают высокий уровень на выходе (более половины величины напряжения питания постоянного тока) и наоборот. Первый буферный усилитель воспроизводит входной сигнал и обеспечивает небольшое выходное сопротивление, предотвращая тем самым воздействие на источник сигнала RС-цепи (вопрос о нагрузке схемы мы рассмотрели в разд. 1.05). Согласно характеристике RС-цепи, выходной сигнал для нее задерживается относительно входного, поэтому выходной буферный усилитель переключается на 10 мкс позже скачка напряжения на входе (напряжение на выходе RС-цепи достигает 50 % своего максимального значения через 0,7RC.

На практике приходится принимать во внимание отклонение входного порога буфера от величины, равной половине напряжения питания, так как это отклонение изменяет задержку и ширину выходного импульса. Иногда подобную схему используют для того, чтобы задержать импульс на время, в течение которого может произойти какое-либо событие. При проектировании схем лучше не прибегать к подобным трюкам, но иногда они бывают полезны.

1.14. Дифференцирующие цепи

Рассмотрим схему, изображенную на рис. 1.36.

Рис. 1.36.

Напряжение на конденсаторе С равно Uвх — U, поэтому

I = Cd(Uвх U)/dt = U/R.

Если резистор и конденсатор выбрать так, чтобы сопротивление R и емкость С были достаточно малыми и выполнялось условие dU/dt << dUвх/dt, то

C(dUвх/dt) = U/R или U(t)RC[dUвх(t)/dt].

Таким образом, мы получили, что выходное напряжение пропорционально скорости изменения входного сигнала.

Для того чтобы выполнялось условие dU/dt << dUвх/dt, произведение RC должно быть небольшим, но при этом сопротивление R не должно быть слишком малым, чтобы не «нагружать» вход (при скачке напряжения на входе изменение напряжения на конденсаторе равно нулю и R представляет собой нагрузку со стороны входа схемы). Более точный критерий выбора для R и С мы получим, когда изучим частотные характеристики. Если на вход схемы подать прямоугольный сигнал, то сигнал на выходе будет иметь вид, представленный на рис. 1.37.

Рис. 1.37. Выходной сигнал (верхний), снимаемый с дифференциатора, на вход которого подается прямоугольный сигнал.

Дифференцирующие цепи удобно использовать для выделения переднего и заднего фронтов импульсных сигналов, и в цифровых схемах можно иногда встретить цепи, подобные той, которая показана на рис. 1.38.

Рис. 1.38. Выделение переднего фронта импульса.

Дифференцирующая RC-цепь генерирует импульсы в виде коротких пиков в моменты переключения входного сигнала, а выходной буферный усилитель преобразует эти импульсы в короткие прямоугольные импульсы. В реальных схемах отрицательный пик бывает небольшим благодаря встроенному в буфер диоду (речь об этом элементе пойдет в разд. 1.25).

Паразитная емкостная связь. Иногда схема неожиданно начинает проявлять дифференцирующие свойства, причем в ситуациях, где они совершенно нежелательны. При этом можно наблюдать сигналы, подобные показанным на рис. 1.39.

Рис. 1.39.

Первый сигнал (а точнее, импульсная помеха) может возникнуть при наличии емкостной связи между рассматриваемой линией и схемой, в которой присутствует прямоугольный сигнал; причиной появления подобной помехи может служить отсутствие оконечного резистора в линии. Если же резистор есть, то следует либо уменьшить сопротивление источника сигналов для линии, либо найти способ ослабления емкостной связи с источником сигналов прямоугольной формы. Сигнал второго типа можно наблюдать в цепи, по которой должен проходить сигнал прямоугольной формы, при наличии дефекта в контакте с этой цепью, например, в щупе осциллографа. Небольшая емкость, возникающая при плохом контакте, и входное сопротивление осциллографа образуют дифференцирующую цепь. Если вы обнаружили, что ваша схема «что-то» дифференцирует, то сказанное может помочь вам найти причину неисправности и устранить ее.

1.15. Интегрирующие цепи

Рассмотрим схему, изображенную на рис. 1.40.

Рис. 1.40.

Напряжение на резисторе R равно Uвх — U, следовательно, I = C(dU/dt) = (UвхU)/R. Если обеспечить выполнение условия U << Uвх за счет большого значения произведения RC, то получим C(dU/dt) ~= Uвх/R или

Мы получили, что схема интегрирует входной сигнал во времени! Рассмотрим, каким образом эта схема обеспечивает аппроксимацию интегрирования в случае входного сигнала прямоугольной формы: U(t) представляет собой знакомый уже нам график экспоненциальной зависимости, определяющей заряд конденсатора (рис. 1.41).

Рис. 1.41.

Первый участок экспоненты (интеграл от почти постоянной величины) — прямая с постоянным углом наклона; при увеличении постоянной времени RC используется все меньший участок экспоненты, тем самым обеспечивается лучшая аппроксимация идеального пилообразного сигнала.

Отметим, что условие U << Uвх равносильно тому, что ток пропорционален напряжению Uвх. Если бы в качестве входного сигнала выступал ток I(t), а не напряжение, то мы получили бы идеальный интегратор. Источником тока может служить резистор с большим сопротивлением и с большим падением напряжения на нем, и на практике часто пользуются этим приближением.

В дальнейшем, когда мы познакомим вас с операционными усилителями и обратной связью, вы узнаете, как построить интегратор, не прибегая к условию Uвых << Uвх. Такой интегратор работает в широком диапазоне частот и напряжений с пренебрежимо малой ошибкой.

Интегрирующие цепи находят широкое применение в аналоговой технике. Их используют в управляющих системах, схемах с обратной связью, при аналого-цифровом преобразовании и генерации колебаний.

Генераторы пилообразного сигнала. Теперь вы без труда разберетесь в том, как работает генератор пилообразного сигнала. Эта схема хорошо зарекомендовала себя и нашла очень широкое применение: ее используют во время-задающих схемах, в генераторах синусоидальных и других типов колебаний, в схемах развертки осциллографов, в аналого-цифровых преобразователях. Схема использует постоянный ток для заряда конденсатора (рис. 1.42).

Рис. 1.42. Источник постоянного тока, заряжающий конденсатор, генерирует напряжение в виде линейно-меняющегося сигнала.

Из уравнения для тока, протекающего через конденсатор, I = C(dU/dt) получим U(t) = (I/C)t. Выходной сигнал изображен на рис. 1.43.

Рис. 1.43.

Линейное нарастание сигнала прекращается тогда, когда «иссякает» напряжение источника тока, т. е. достигается его предельное значение. Кривая для простой RC-цепи с резистором, подключенным к источнику напряжения, ведет себя аналогично случаю достижения предела источником тока. На рис. 1.43 эта вторая кривая показана для случая, когда R выбрано так, чтобы ток при нулевом выходном напряжении был равен току источника тока; при этом вторая кривая стремится к тому же пределу, что и ломаная. (В реальных источниках тока выходное напряжение ограничено напряжением используемых в них источников питания, так что такое поведение вполне правдоподобно.) В следующей главе, посвященной транзисторам, мы построим простые схемы источников тока, а в главах, где рассматриваются операционные усилители и полевые транзисторы, — их усовершенствованные типы.

Вот как много интересных вопросов ожидает нас впереди.

Упражнение 1.15. Ток 1 мА заряжает конденсатор емкостью 1 мкФ. Через какое время напряжение достигнет 10 В?

Определение электроемкости конденсатора в цепи переменного тока

ЛАБОРАТОРНАЯ РАБОТА № 23

ОПРЕДЕЛЕНИЕ ЭЛЕКТРОЕМКОСТИ КОНДЕНСАТОРА В ЦЕПИ ПЕРЕМЕННОГО ТОКА

Цель работы: определение неизвестной электроемкости конденсатора в цепи переменного тока, проверка законов последовательного и параллельного соединения конденсаторов.

Приборы и принадлежности: источник переменного тока, батарея конденсаторов, миллиамперметр, вольтметр.

Методические указания

Между потенциалом проводника ? и зарядом q, сосредоточенном на нем, существует прямопропорциональная зависимость (q ~ φ):

                                                                                                   (1)

Коэффициент пропорциональности «С» является характеристикой проводника и называется электроемкостью. Из уравнения (1) следует, что

                                                                                              (2)

т.е. электроемкость численно равна заряду, который необходимо сообщить проводнику, чтобы увеличить его потенциал на единицу.

Электроемкость уединенного проводника зависит от его геометрических размеров, формы и диэлектрической проницаемости окружающей среды и не зависит от материала проводника и заряда, сосредоточенного на нем. Например, электроемкость уединенного шара радиусом R, находящегося в среде с диэлектрической проницаемостью ? равна (в системе СИ):

                                                                                   (3)

Единица измерения электроемкости следует из определения (2) и в системе СИ равна:

             

Называется она Фарада. Т.к. Фарада очень крупная единица, то на практике электроемкость проводников выражают в долях Фарады — микрофарадах и пикофарадах.

Электроемкость проводника зависит также от наличия вблизи него других проводников, а именно: если в поле заряженного проводника внести незаряженный, то в последнем произойдет разделение зарядов (электризация) и индуцированное, таким образом, поле будет накладываться на внешнее и ослаблять его. В результате этого потенциал его уменьшится, а электроемкость, согласно формуле (2) возрастет, поскольку заряд проводника остается неизменным.

Характеристикой системы, состоящей из нескольких проводников, является взаимная электроемкость. В случае двух проводников она численно равна заряду q, который необходимо перенести из одного проводника на другой, чтобы изменить разность потенциалов на единицу (?φ = U):

          

  Взаимная электроемкость системы проводников больше, чем электроемкость каждого из них в отдельности. Система из двух (или более) проводников — обкладок — разделенных слоем диэлектрика, называется конденсатором. На обкладках заряженного конденсатора сосредотачиваются равные по величине, но противоположные по знаку заряды. Обкладкам придают такую форму, и они расположены так близко друг к другу, что электрическое поле оказывается сосредоточенным между ними. Поэтому на электроемкость конденсатора не сказывается наличие вблизи него других проводников.

Электроемкость конденсатора зависит от его формы, геометрических размеров диэлектрической проницаемости диэлектрика. Наиболее распространены плоские и цилиндрические конденсаторы, электроемкости которых соответственно равны:

                                                                   (4)

где S — площадь обкладок, d — расстояние между ними в плоском конденсаторе, r1 и r2 — радиусы внутреннего и внешнего цилиндров цилиндрического конденсатора, l — его длина.

Конденсаторы можно последовательно или параллельно соединять между собой.

При последовательном соединении:

                                                                               (5)

При параллельном соединении:

                                                                               (6)

При подключении конденсатора к источнику постоянного тока на каждой обкладке его скапливается заряд q, а между ними создается электрическое поле, энергия которого равна:

                                                          (7)

После того, как пластины приобретут потенциалы, равные потенциалам полюсов источников тока, и конденсатор окажется заряженным, ток в цепи прекратится.

Иначе обстоит дело в цепи переменного тока. В этом случае происходит непрерывная перезарядка конденсатора, и включение его в такую цепь равносильно введению добавочного сопротивления. Рассмотрим этот процесс более подробно.

Пусть на обкладки конденсатора подано переменное напряжение:

                                                                                  (8)

где ? — круговая частота, связанная с периодом T и частотой переменного тока ? соотношением ?=2π / T=2π ν.

Мгновенное значение заряда на обкладках равно

                                                (9)

Тогда для мгновенного значения силы тока имеем:

                    (10)

 Из формул (8) и (10) следует, что ток опережает по фазе напряжение на ?/2 (четверть периода), т.е. тогда, когда разность потенциалов на обкладках максимальная, ток минимален и наоборот. Графически зависимость тока в цепи конденсатора и напряжения на нем от времени изображена на рис .1.

 

Uo                                                                           Io   

 

                                                                                       t 

                                  T/2          T

                                            Рис. 1

Из формулы (10) для амплитудного значения силы тока получим

                                                                      (11)

Из сравнения этого выражения с законом Ома 

                                               

следует, что величина 1/ωΡ играет роль сопротивления и называется поэтому емкостным сопротивлением в цепи переменного тока Rс:

                            (12)

Емкостное сопротивление относится к классу реактивных сопротивлений, т.е. таких, при которых не происходит рассеяния энергии в виде выделения теплоты. Энергия на реактивном сопротивлении накапливается в течение 1/4 периода при создании электрического поля, а затем в следующей четверти периода при уменьшении поля она возвращается источнику. Механическим аналогом такого сопротивления является пружина, где внешняя энергия при сжатии переходит в потенциальную энергию и наоборот.

Так из формулы (12) следует, что измерив емкостное сопротивление Rc и зная частоту переменного тока, можно найти электроемкость конденсатора:

                                                     (13)

Порядок выполнения работы

1.  Собрать электрическую цепь согласно рис. 2, где U — источник переменного тока, БК — батарея конденсаторов.

 

               

                                                                       Рис. 2

2.  Поставить переключатель батареи конденсаторов в положение С1. Включить источник тока и измерить силу тока при четырех различных напряжениях.

3.  По закону Ома (12) вычислить значение Rc для каждого измерения. Найти среднее значение Rc и ?Rc.

4.  По формуле (13), используя средние значения Rc и ?Rc, вычислить значения С1 и ?С1. Частота переменного тока ? равна 50,0±0,1 Гц. Данные измерений и вычислений занести в таблицу.

U

I

Rc

ΔRc

C

ΔC

1

2

3

4

Ср.

1.  Аналогичным образом найти С2, Спосл., Спарал., поставив переключатель батареи конденсаторов соответственно положения С2, Спосл., Спарал.

2.  По найденным значениям С1,С2, а также  формулам (5) и (6) вычислить теоретическое значение Спосл. и Спарал. и сравнить их с измеренными.

Контрольные вопросы

1.  Что называется электроемкостью проводника и от чего она зависит?

2.  Что называется электроемкостью проводников?

3.  Что представляет собой конденсатор?

4.  Чему равна электроемкость батареи конденсаторов при последовательном и параллельном их соединении?

5.  Какой запас энергии сосредоточен в конденсаторе?

6.  Что называется емкостным сопротивлением, от чего оно зависит и какова его характерная особенность?

Литература

1.  Савельев И.В. Курс общей физики. Т.2.

2.  Зисман Т.А., Тодес О.М. Курс общей физики. Т.II, гл.3.

3.  Грабовский Р.И. Курс физики.

Конденсатор в цепи постоянного тока для чего

На чтение 18 мин. Опубликовано

При подключении конденсатора к источнику постоянного тока под действием электрического поля на нижнюю обкладку движутся электроны. В следствии, явления электростатической индукции с верхней обкладки конденсатора заряды уходят к положительному выводу источника питания в цепи возникает токток заряда по мере накопления зарядов в конденсаторе, растёт напряжение , а ток заряда уменьшается, и так, – конденсатор подключённый к источнику тока, заряжается до Uист .

Конденсатор в цепи постоянного тока

Кратковременный ток в цепи называется ток заряда, а так как он существует короткое время, то говорят, конденсатор постоянный ток не пропускает.

Считается что конденсатор заряжается если напряжение на нём составляет 0,63 от Uист и это происходит за время
равное Τ

Τ заряда – постоянная времени заряда конденсатора в секундах

Одна секунда – 1с = 10 3 мс = 10 6 мкс =10 12 нс

Rзар – сопротивление в Омах

График заряда конденсатора

Работа конденсатора в цепи постоянного тока

Считается, что конденсатор разрядится если напряжение на нём составляет 0,37 от напряжения источника и это происходит за время Τ разряда.

Во всех радиотехнических и электронных устройствах кроме транзисторов и микросхем применяются конденсаторы. В одних схемах их больше, в других меньше, но совсем без конденсаторов не бывает практически ни одной электронной схемы.

При этом конденсаторы могут выполнять в устройствах самые разные задачи. Прежде всего, это емкости в фильтрах выпрямителей и стабилизаторов. С помощью конденсаторов передается сигнал между усилительными каскадами, строятся фильтры низких и высоких частот, задаются временные интервалы в выдержках времени и подбирается частота колебаний в различных генераторах.

Свою родословную конденсаторы ведут от лейденской банки, которую в середине XVIII века в своих опытах использовал голландский ученый Питер ван Мушенбрук. Жил он в городе Лейдене, так что нетрудно догадаться, почему так называлась эта банка.

Собственно это и была обыкновенная стеклянная банка, выложенная внутри и снаружи оловянной фольгой – станиолем. Использовалась она в тех же целях, как и современная алюминиевая, но тогда алюминий открыт еще не был.

Единственным источником электричества в те времена была электрофорная машина, способная развивать напряжение до нескольких сотен киловольт. Вот от нее и заряжали лейденскую банку. В учебниках физики описан случай, когда Мушенбрук разрядил свою банку через цепь из десяти гвардейцев взявшихся за руки.

В то время никто не знал, что последствия могут быть трагическими. Удар получился достаточно чувствительным, но не смертельным. До этого не дошло, ведь емкость лейденской банки была незначительной, импульс получился очень кратковременным, поэтому мощность разряда была невелика.

Как устроен конденсатор

Устройство конденсатора практически ничем не отличается от лейденской банки: все те же две обкладки, разделенные диэлектриком. Именно так на современных электрических схемах изображаются конденсаторы. На рисунке 1 показано схематичное устройство плоского конденсатора и формула для его расчета.

Рисунок 1. Устройство плоского конденсатора

Здесь S – площадь пластин в квадратных метрах, d – расстояние между пластинами в метрах, C — емкость в фарадах, ε – диэлектрическая проницаемость среды. Все величины, входящие в формулу, указаны в системе СИ. Эта формула справедлива для простейшего плоского конденсатора: можно просто расположить рядом две металлические пластины, от которых сделаны выводы. Диэлектриком может служить воздух.

Из этой формулы можно понять, что емкость конденсатора тем больше, чем больше площадь пластин и чем меньше расстояние между ними. Для конденсаторов с другой геометрией формула может быть иной, например, для емкости одиночного проводника или электрического кабеля. Но зависимость емкости от площади пластин и расстояния между ними та же, что и у плоского конденсатора: чем больше площадь и чем меньше расстояние, тем больше емкость.

На самом деле пластины не всегда делаются плоскими. У многих конденсаторов, например металлобумажных, обкладки представляют собой алюминиевую фольгу свернутую вместе с бумажным диэлектриком в плотный клубок, по форме металлического корпуса.

Для увеличения электрической прочности тонкая конденсаторная бумага пропитывается изолирующими составами, чаще всего трансформаторным маслом. Такая конструкция позволяет делать конденсаторы с емкостью до нескольких сотен микрофарад. Примерно так же устроены конденсаторы и с другими диэлектриками.

Формула не содержит никаких ограничений на площадь пластин S и расстояние между пластинами d. Если предположить, что пластины можно развести очень далеко, и при этом площадь пластин сделать совсем незначительной, то какая-то емкость, пусть небольшая, все равно останется. Подобное рассуждение говорит о том, что даже просто два проводника, расположенные по соседству, обладают электрической емкостью.

Этим обстоятельством широко пользуются в высокочастотной технике: в некоторых случаях конденсаторы делаются просто в виде дорожек печатного монтажа, а то и просто двух скрученных вместе проводков в полиэтиленовой изоляции. Обычный провод–лапша или кабель также обладают емкостью, причем с увеличением длины она увеличивается.

Кроме емкости C, любой кабель обладает еще и сопротивлением R. Оба этих физических свойства распределены по длине кабеля, и при передаче импульсных сигналов работают как интегрирующая RC – цепочка, показанная на рисунке 2.

На рисунке все просто: вот схема, вот входной сигнал, а вот он же на выходе. Импульс искажается до неузнаваемости, но это сделано специально, для чего и собрана схема. Пока же речь идет о влиянии емкости кабеля на импульсный сигнал. Вместо импульса на другом конце кабеля появится вот такой «колокол», а если импульс короткий, то он может и вовсе не дойти до другого конца кабеля, вовсе пропасть.

Исторический факт

Здесь вполне уместно вспомнить историю о том, как прокладывали трансатлантический кабель. Первая попытка в 1857 году потерпела неудачу: телеграфные точки – тире (прямоугольные импульсы) искажались так, что на другом конце линии длиной 4000 км разобрать ничего не удалось.

Вторая попытка была предпринята в 1865 году. К этому времени английский физик У. Томпсон разработал теорию передачи данных по длинным линиям. В свете этой теории прокладка кабеля оказалась более удачной, сигналы принять удалось.

За этот научный подвиг королева Виктория пожаловала ученого рыцарством и титулом лорда Кельвина. Именно так назывался небольшой город на побережье Ирландии, где начиналась прокладка кабеля. Но это просто к слову, а теперь вернемся к последней букве в формуле, а именно, к диэлектрической проницаемости среды ε.

Немножко о диэлектриках

Эта ε стоит в знаменателе формулы, следовательно, ее увеличение повлечет за собой возрастание емкости. Для большинства используемых диэлектриков, таких как воздух, лавсан, полиэтилен, фторопласт эта константа практически такая же, как у вакуума. Но вместе с тем существует много веществ, диэлектрическая проницаемость которых намного выше. Если воздушный конденсатор залить ацетоном или спиртом, то его емкость возрастет раз в 15…20.

Но подобные вещества обладают кроме высокой ε еще и достаточно высокой проводимостью, поэтому такой конденсатор заряд держать будет плохо, он быстро разрядится сам через себя. Это вредное явление называется током утечки. Поэтому для диэлектриков разрабатываются специальные материалы, которые позволяют при высокой удельной емкости конденсаторов обеспечивать приемлемые токи утечки. Именно этим и объясняется такое разнообразие видов и типов конденсаторов, каждый из которых предназначен для конкретных условий.

Электролитический конденсатор

Наибольшей удельной емкостью (соотношение емкость / объем) обладают электролитические конденсаторы. Емкость «электролитов» достигает до 100 000 мкФ, рабочее напряжение до 600В. Такие конденсаторы работают хорошо только на низких частотах, чаще всего в фильтрах источников питания. Электролитические конденсаторы включаются с соблюдением полярности.

Электродами в таких конденсаторах является тонкая пленка из оксида металлов, поэтому часто эти конденсаторы называют оксидными. Тонкий слой воздуха между такими электродами не очень надежный изолятор, поэтому между оксидными обкладками вводится слой электролита. Чаще всего это концентрированные растворы кислот или щелочей.

На рисунке 3 показан один из таких конденсаторов.

Рисунок 3. Электролитический конденсатор

Чтобы оценить размеры конденсатора рядом с ним сфотографировался простой спичечный коробок. Кроме достаточно большой емкости на рисунке можно разглядеть еще и допуск в процентах: ни много ни мало 70% от номинальной.

В те времена, когда компьютеры были большими и назывались ЭВМ, такие конденсаторы стояли в дисководах (по-современному HDD). Информационная емкость таких накопителей теперь может вызвать лишь улыбку: на двух дисках диаметром 350 мм хранилось 5 мегабайт информации, а само устройство весило 54 кг.

Основным назначением показанных на рисунке суперконденсаторов был вывод магнитных головок из рабочей зоны диска при внезапном отключении электроэнергии. Такие конденсаторы могли хранить заряд несколько лет, что было проверено на практике.

Чуть ниже с электролитическими конденсаторами будет предложено проделать несколько простых опытов, чтобы понять, что может делать конденсатор.

Для работы в цепях переменного тока выпускаются неполярные электролитические конденсаторы, вот только достать их почему-то очень непросто. Чтобы как-то эту проблему обойти, обычные полярные «электролиты» включают встречно-последовательно: плюс-минус-минус-плюс.

Если полярный электролитический конденсатор включить в цепь переменного тока, то сначала он будет греться, а потом раздастся взрыв. Отечественные старые конденсаторы разлетались во все стороны, импортные же имеют специальное приспособление, позволяющее избежать громких выстрелов. Это, как правило, либо крестовая насечка на донышке конденсатора, либо отверстие с резиновой пробкой, расположенное там же.

Очень не любят электролитические конденсаторы повышенного напряжения, даже если полярность соблюдена. Поэтому никогда не надо ставить «электролиты» в цепь, где предвидится напряжение близкое к максимальному для данного конденсатора.

Иногда в некоторых, даже солидных форумах, начинающие задают вопрос: «На схеме означен конденсатор 470µF * 16V, а у меня есть 470µF * 50V, можно ли его поставить?». Да, конечно можно, вот обратная замена недопустима.

Конденсатор может накапливать энергию

Разобраться с этим утверждением поможет простая схема, показанная на рисунке 4.

Рисунок 4. Схема с конденсатором

Главным действующим лицом этой схемы является электролитический конденсатор C достаточно большой емкости, чтобы процессы заряда – разряда протекали медленно, и даже очень наглядно. Это дает возможность наблюдать работу схемы визуально с помощью обычной лампочки от карманного фонаря. Фонари эти давно уступили место современным светодиодным, но лампочки для них продаются до сих пор. Поэтому, собрать схему и провести простые опыты очень даже просто.

Может быть, кто-то скажет: «А зачем? Ведь и так все очевидно, да если еще и описание почитать…». Возразить тут, вроде, нечего, но любая, даже самая простая вещь остается в голове надолго, если ее понимание пришло через руки.

Итак, схема собрана. Как она работает?

В положении переключателя SA, показанном на схеме, конденсатор C заряжается от источника питания GB через резистор R по цепи: +GB __ R __ SA __ C __ -GB. Зарядный ток на схеме показан стрелкой с индексом iз. Процесс заряда конденсатора показан на рисунке 5.

Рисунок 5. Процесс заряда конденсатора

На рисунке видно, что напряжение на конденсаторе возрастает по кривой линии, в математике называемой экспонентой. Ток заряда прямо-таки зеркально отражает напряжение заряда. По мере того, как напряжение на конденсаторе растет, ток заряда становится все меньше. И только в начальный момент соответствует формуле, показанной на рисунке.

Через некоторое время конденсатор зарядится от 0В до напряжения источника питания, в нашей схеме до 4,5В. Весь вопрос в том, как это время определить, сколько ждать, когда же конденсатор зарядится?

Постоянная времени «тау» τ = R*C

В этой формуле просто перемножаются сопротивление и емкость последовательно соединенных резистора и конденсатора. Если, не пренебрегая системой СИ, подставить сопротивление в Омах, емкость в Фарадах, то результат получится в секундах. Именно это время необходимо для того, чтобы конденсатор зарядился до 36,8% напряжения источника питания. Соответственно для заряда практически до 100% потребуется время 5* τ.

Часто, пренебрегая системой СИ, подставляют в формулу сопротивление в Омах, а емкость в микрофарадах, тогда время получится в микросекундах. В нашем случае результат удобнее получить в секундах, для чего придется микросекунды просто умножить на миллион, а проще говоря, переместить запятую на шесть знаков влево.

Для схемы, показанной на рисунке 4, при емкости конденсатора 2000мкФ и сопротивлении резистора 500Ω постоянная времени получится τ = R*C = 500 * 2000 = 1000000 микросекунд или ровно одна секунда. Таким образом, придется подождать приблизительно 5 секунд, пока конденсатор зарядится полностью.

Если по истечении указанного времени переключатель SA перевести в правое положение, то конденсатор C разрядится через лампочку EL. В этот момент получится короткая вспышка, конденсатор разрядится и лампочка погаснет. Направление разряда конденсатора показано стрелкой с индексом iр. Время разряда также определяется постоянной времени τ. График разряда показан на рисунке 6.

Рисунок 6. График разряда конденсатора

Конденсатор не пропускает постоянный ток

Убедиться в этом утверждении поможет еще более простая схема, показанная на рисунке 7.

Рисунок 7. Схема с конденсатором в цепи постоянного тока

Если замкнуть переключатель SA, то последует кратковременная вспышка лампочки, что свидетельствует о том, что конденсатор C зарядился через лампочку. Здесь же показан и график заряда: в момент замыкания переключателя ток максимальный, по мере заряда конденсатора уменьшается, а через некоторое время прекращается совсем.

Если конденсатор хорошего качества, т.е. с малым током утечки (саморазряда) повторное замыкание выключателя к вспышке не приведет. Для получения еще одной вспышки конденсатор придется разрядить.

Конденсатор в фильтрах питания

Конденсатор ставится, как правило, после выпрямителя. Чаще всего выпрямители делаются двухполупериодными. Наиболее распространенные схемы выпрямителей показаны на рисунке 8.

Рисунок 8. Схемы выпрямителей

Однополупериодные выпрямители также применяются достаточно часто, как правило, в тех случаях, когда мощность нагрузки незначительна. Самым ценным качеством таких выпрямителей является простота: всего один диод и обмотка трансформатора.

Для двухполупериодного выпрямителя емкость конденсатора фильтра можно рассчитать по формуле

C = 1000000 * Po / 2*U*f*dU, где C емкость конденсатора мкФ, Po мощность нагрузки Вт, U напряжение на выходе выпрямителя В, f частота переменного напряжения Гц, dU амплитуда пульсаций В.

Большое число в числителе 1000000 переводит емкость конденсатора из системных Фарад в микрофарады. Двойка в знаменателе представляет собой число полупериодов выпрямителя: для однополупериодного на ее месте появится единица

C = 1000000 * Po / U*f*dU,

а для трехфазного выпрямителя формула примет вид C = 1000000 * Po / 3*U*f*dU.

Суперконденсатор – ионистор

В последнее время появился новый класс электролитических конденсаторов, так называемый ионистор. По своим свойствам он похож на аккумулятор, правда, с несколькими ограничениями.

Заряд ионистора до номинального напряжения происходит в течение короткого времени, буквально за несколько минут, поэтому его целесообразно использовать в качестве резервного источника питания. По сути ионистор прибор неполярный, единственное, чем определяется его полярность это зарядкой на заводе – изготовителе. Чтобы в дальнейшем эту полярность не перепутать она указывается знаком +.

Большую роль играют условия эксплуатации ионисторов. При температуре 70˚C при напряжении 0,8 от номинального гарантированная долговечность не более 500 часов. Если же прибор будет работать при напряжении 0,6 от номинального, а температура не превысит 40 градусов, то исправная работа возможна в течение 40 000 часов и более.

Наиболее распространенное применение ионистора это источники резервного питания. В основном это микросхемы памяти или электронные часы. В этом случае основным параметром ионистора является малый ток утечки, его саморазряд.

Достаточно перспективным является использование ионисторов совместно с солнечными батареями. Здесь также сказывается некритичность к условию заряда и практически неограниченное число циклов заряд-разряд. Еще одно ценное свойство в том, что ионистор не нуждается в обслуживании.

Пока получилось рассказать, как и где работают электролитические конденсаторы, причем, в основном в цепях постоянного тока. О работе конденсаторов в цепях переменного тока будет рассказано в другой статье — Конденсаторы для электроустановок переменного тока.

Практически во всех электронных устройствах, от самых простых до высокотехнологичных, таких как материнские платы компьютеров, можно встретить один неизменно присутствующий элемент, являющийся пассивным компонентом. Но к сожалению, мало кто знает как устроен и для чего нужен конденсатор, и какие виды этого накопителя бывают.

Просто о сложном

Итак, это небольшое устройство для накопления электрического поля или заряда похоже на обычную банку, ту, в которой маринуют помидоры или хранят муку. Она точно так же в себе накапливает сухое вещество или жидкость, которую в неё поместят. Аналогия проста: по цепи бегут электроны, а на своей дороге встречают проводников, которые ведут их в «банку», где они и накапливаются, усиливая заряд.

Для того чтобы выяснить, много ли элекрончиков так можно собрать, и в какой момент накопление прекратится (банка лопнет), электрический процесс обычно сравнивают с водопроводом. Если представить трубу, в которой течёт вода, закачиваемая туда насосом, то где-то в центре трубопровода нужно вообразить мягкую мембрану, растягивающуюся под давлением жидкости. Очевидно, что она будет растягиваться до определённого предела, пока не разорвётся или, если попалась очень крепкая, не уравновесит силу насоса.

Такой пример показывает, как работает конденсатор, только мембрана заменяется электрическим полем, которое увеличивается по мере зарядки накопителя (работы насоса), уравновешивая напряжение источника питания. Очевидно, что этот процесс не бесконечный, и предельный заряд существует, по достижении которого «банка» выйдет из строя и перестанет выполнять свои функции.

Устройство и принцип работы

Конденсатор — устройство, состоящее из двух пластин (обкладок), имеющих между собой пустоту. Напряжение к нему подаётся через проводки, подсоединённые к пластинкам. Современные приборы, по сути, не сильно отличаются от макетов на уроках физики, они также состоят из диэлектрика и обкладок. Следует отметить, что именно вещество или его отсутствие (вакуум), плохо проводящее электричество, изменяет характеристики накопителя.

Суть принципа работы конденсатора проста: дали напряжение, и заряд начал накапливаться. Для примера следует рассмотреть как ведёт себя накопитель в двух вариантах электрической цепи:

  • Постоянный ток. Если в цепь с подключённым к ней конденсатором подать ток, то можно увидеть, что стрелка на амперметре начнёт двигаться, а потом быстро вернётся в исходное положение. Это объясняется просто: устройство быстро зарядилось, то есть источник питания был уравновешен обкладками накопителя, и тока не стало. Поэтому часто говорят, что в условиях постоянного тока конденсатор не работает. Такое утверждение неправильное, всё функционирует, но очень непродолжительное время.
  • Переменный ток — это когда электроны двигаются сначала в одну, а затем в другую сторону. Если представить такую цепь с подключённым к ней накопителем, то на обеих обкладках конденсатора будут попеременно накапливаться положительные и отрицательные заряды. Это говорит о том, что переменный ток свободно протекает через устройство.

Поскольку конденсатор задерживает постоянный ток, но пропускает переменный, отсюда формируются и сферы его назначения, например, для устройств, в которых нужно убрать постоянную составляющую в сигнале. Вполне очевидно, что накопитель обладает сопротивлением, а вот мощность на нём не выделяется, поэтому он не греется.

Основные виды

Рядовой пользователь не всегда знает о том, каким конденсатором снабжено его устройство. А ведь каждый вид имеет свои недостатки и преимущества, а также эксплуатационные особенности. Существуют две большие группы этих устройств, предназначенные для электрической цепи с переменным и постоянным током. Но всё-таки основная классификация ведётся по типу диэлектрика, который находится между облатками конденсатора. Основные виды:

  • Керамические. Имеют маленький размер, малый ток утечки и небольшую индуктивность. Отлично работают в условиях высоких частот, в цепях пульсирующего, постоянного и переменного тока. Представлены в различном диапазоне напряжений и ёмкостей, в зависимости от того, для чего конденсатор предназначен.
  • Слюдяные. В настоящее время почти не используются и не выпускаются. В накопителях такого типа диэлектриком служит слюда. Рабочее напряжение таких конденсаторов в диапазоне — 200−1500 В.
  • Бумажные. В алюминиевых облатках заключена конденсаторная бумага. Выдерживают напряжение 160−1500 В.
  • Полиэстеровые. Максимальная ёмкость не превышает 15 мФ, рабочее напряжение — 50−1500 В.
  • Полипропиленовые. Выгодно выделяются на фоне остальных собратьев двумя преимуществами. Первое — маленький допуск ёмкости (+/- 1%), второе — до 3 кВ рабочего напряжения.

Отдельно стоит отметить электролитические конденсаторы. Главное их отличие от других видов — подключения только к цепи постоянного или пульсирующего тока. Такие накопители имеют полярность — это особенность их конструкции, поэтому неправильное подключение ведёт к вздутию или взрыву устройства. Они обладают большой ёмкостью, что делает конденсатор электролитический пригодным для применения в выпрямительных цепях.

Сферы применения

Можно смело сказать, что конденсаторы используют практически во всех электронных и радиотехнических схемах. Чтобы иметь представление о том, где и зачем нужен конденсатор, следует вспомнить его способность сохранять заряд и разряжаться в нужное время, а также пропускать переменный ток и не пропускать постоянный. А это значит, что такие устройства используются во многих технических сферах, например:

  • телефонии;
  • в производстве счётных и запоминающих устройств;
  • автоматике;
  • при создании измерительных приборов и многих других.

Электрические накопители можно встретить как в телевизорах, так и в приборах радиолокации, где необходимо формировать импульс большой мощности, для чего и служит конденсатор. Невозможно встретить блок питания без этих устройств или сетевой фильтр.

Нужно сказать, что накопители применяют и в сферах, не связанных с электрикой, например, в производстве металла и добыче угля, где используют конденсаторные электровозы.

Конденсатор в качестве сопротивления в сети переменного тока

Известно, что конденсатор, установленный в цепи переменного тока, обладает сопротивлением, зависящим от частоты, и называется реактивным.

Используя его, можно также гасить излишнее напряжение сети, причем мощность на реактивном сопротивлении не выделяется, что является большим преимуществом конденсатора перед гасящим резистором.

Так как полное сопротивление 2 цепи, составленной из последовательно включенных нагрузки с активным сопротивлением и конденсатора с реактивным сопротивлением Хс, равно:

то непосредственный расчет емкости гасящего конденсатора довольно сложен.

Для определения ее проще пользоваться номограммой, приведенной на рис. 1. На номограмме по оси абсцисс отложены сопротивления в кОм, по оси ординат — емкость С гасящих конденсаторов в мкФ и по оси, проведенной под углом 45° к оси абсцисс,— полные сопротивления 2 цепи в кОм.

Чтобы воспользоваться номограммой, предварительно нужно по закону Ома или по формуле мощности определить Rн и Z. На оси абсцисс номограммы находят вычисленное значение и проводят из этой точки вертикальную прямую, параллельную оси ординат. Затем на наклонной оси отыскивают ранее определенное значение 2.

Рис. 1. Номограмма.

Рис. 2. Пример включения конденсатора.

Из точки начала координат через точку 2 проводят дугу, которая должна пересечь линию, проведенную параллельно оси ординат. Из точки пересечения ведут линию, параллельную оси абсцисс. Точка, где эта линия встретится с осью ординат, укажет искомую емкость гасящего конденсатора.

Пример 1. Определить емкость конденсатора, который нужно соединить последовательно с электропаяльником 127 В, 25 Вт, чтобы его можно было включить в сеть переменного тока напряжением 220 В (рис. 2, а).

Находим

где U — напряжение, на которое рассчитан электропаяльник, Р — мощность электропаяльника.

Чтобы определить 2, нужно знать ток I, протекающий в цепи:

Тогда Z равно:

Как найти емкость гасящего конденсатора, пользуясь вычисленными предварительными данными, показано на номограмме жирными линиями.

Пример 2. Мостовой выпрямитель (рис. 2,6) с выходным напряжением (Uвых — 18 В и током нагрузки Iн = 20 мА необходимо питать от сети напряжением 127 В. Найти емкость конденсатора С1, который нужно подключить последовательно с выпрямителем, чтобы погасить излишнее напряжение.

Определяем сопротивление нагрузки:

и полное сопротивление цепи:

Далее определяют емкость гасящего конденсатора С/ по номограмме. Как это сделать, показано на ней пунктиром. Результат, полученный по расчету (0,51 мкФ), можно округлить до 0,5 мкФ.

Для гашения напряжения можно использовать только бумажные конденсаторы (МБМ, МБГП и др.), предназначенные для работы в цепи переменного тока. Их рабочее напряжение для большей надежности работы должно в 2—3 раза превышать напряжение, которое нужно погасить.

Литература: В. Г. Бастанов. 300 практических советов, 1986г.

Электрические конденсаторы в цепях переменного тока

В этом месяце мы обсудим функции конденсатора в цепи переменного тока, включая заряд и разряд, приложения и соединения в силовых цепях, а также безопасность конденсаторов.

Электрический конденсатор — это электрическое устройство, которое накапливает электричество или электрическую энергию и улучшает коэффициент мощности цепи переменного тока. Он состоит из трех основных частей.Две обычно представляют собой металлические пластины, разделенные и изолированные третьей частью, известной как диэлектрик. Заряд конденсатора зависит от размера и расстояния между проводящими пластинами, а также от типа изолирующей или диэлектрической среды между пластинами.

Все конденсаторы, независимо от типа, обозначаются по их зарядной емкости. Для цепей электроприборов, таких как двигатели и разрядное освещение высокой интенсивности, конденсаторы обозначаются фарадом, единицей электрической емкости, названной в честь британского ученого Майкла Фарадея.В распределительной сети конденсаторы обозначаются в киловольт-амперах, реактивных, или кВАр, для простоты применения. Измерители потребления измеряют потребность в коэффициенте мощности в кВАр. Если нагрузка двигателя потребителя вызывает индуктивное напряжение 700 кВАр на линии, это можно исправить, подключив к линии емкостное напряжение 700 кВАр. Это не так просто, но идею вы поняли.

Зарядка и разрядка конденсатора в цепи переменного тока
Конденсатор немного похож на батарею. Хотя они работают совершенно по-разному, конденсаторы и аккумуляторы хранят электрическую энергию.Вы знаете, что батарея имеет две клеммы. Внутри батареи химические реакции производят электроны на одном выводе и поглощают электроны на другом. Конденсатор — гораздо более простое устройство, и он не может производить новые электроны — он только хранит их.

Например, когда вы видите в небе молнию, вы видите огромный конденсатор. Одна пластина — это облако, другая пластина — это земля, а молния — это заряд, высвобождающийся между этими двумя пластинами. Очевидно, что в таком большом конденсаторе вы можете удерживать огромное количество заряда.

Применения в цепях питания
Конденсаторы должны применяться аккуратно и правильно. Как указывалось ранее, конденсатор предназначен для улучшения коэффициента мощности схемы. Конденсатор только корректирует коэффициент мощности конденсатора обратно в систему. Конденсаторы не влияют на коэффициент мощности между конденсатором и реактивной нагрузкой, вызывающей коэффициент мощности. Неправильно установленные конденсаторы могут обеспечивать больший реактивный ток, чем требуется нагрузке, что приводит к опережающему коэффициенту мощности и увеличению потерь вместо уменьшения.Именно поэтому конденсаторы находятся после тщательного изучения системы квалифицированными инженерами.

Соединения в цепях питания
Установка одного конденсатора или батареи конденсаторов является простой процедурой после определения правильного размера и места установки. Во многих отношениях конденсатор установить намного проще, чем трансформатор, потому что нет вторичных вводов, а конденсатор представляет собой герметичный блок. У одних агрегатов две втулки, у других — одна.

Распределительные конденсаторы с двумя изолированными вводами обычно подключаются между фазами, но могут быть соединены фазой с землей. Конденсаторы с одним изолированным вводом обычно соединяются фазой с изолированным вводом, а корпус — с землей. В подстанциях конденсаторы распределительного напряжения подключаются параллельно, разделяя фазные напряжения передачи. Они установлены на изолированном каркасе, который является частью межсоединения, работающего при фазном напряжении передачи. Любой, кто работает с любым конденсатором, должен хорошо знать, как он подключен и при каком напряжении работает.

Вам также следует знать о конденсаторах постоянной и переключаемой емкости. Эти термины просто относятся к способу подачи питания на конденсатор. Если он зафиксирован, проходной изолятор, подключенный к источнику, попадает непосредственно в плавкий вырез. Этот выключатель — единственное средство включения или выключения конденсатора. В случае переключаемого конденсатора проходной изолятор, подключенный к источнику, идет к переключающему устройству, включенному последовательно между плавким предохранителем и конденсатором. Назначение этого переключающего устройства — позволить конденсатору работать в то время, когда это больше всего необходимо.Существует множество типов элементов управления, которые определяют, должен ли конденсатор быть включен или выключен в зависимости от требований системы. Ниже приведены примеры средств управления и то, для чего они чаще всего используются:
• Контроль времени используется в областях, где известно, что нагрузка возникает с определенными интервалами либо из-за промышленного использования, либо из-за требований жилых помещений.
• Текущее управление используется в областях, где нагрузка клиента является прерывистой и не всегда присутствует в одно и то же время дня.
• Контроль температуры используется в тех областях, где сезонные изменения увеличивают индуктивную нагрузку на систему (например,г., кондиционер). В основном доступны два основных контроля температуры: включение при 85 градусах и выключение при 65 градусах или включение при 90 градусах и выключение при 70 градусах.
• Контроль напряжения используется в областях, где нагрузка вызывает падение напряжения в системе, которое можно легко контролировать. Регуляторы напряжения также часто используются вместе с регуляторами времени, тока и температуры.
• Ручное управление используется для более простого отключения переключаемого банка, который часто используется для сезонной нагрузки.

Безопасность конденсаторов
Хотя конденсаторы являются простыми устройствами, они чрезвычайно опасны после отключения от обслуживания, поскольку могут сохранять заряд.Они должны быть построены с резисторами утечки, которые снижают их напряжение до менее 50 вольт через пять минут. Однако никогда не принимайте ничего как должное. Обязательно подождите пять минут после отсоединения, а затем закоротите втулки перемычкой с помощью рукоятки для дробовика. Перед началом работы с любым конденсатором, установленным на опоре, всегда выполняйте эти процедуры, потому что конденсатор может удерживать заряд без каких-либо признаков того, что он это делает.

При выводе конденсаторов из эксплуатации нельзя выделить следующие шаги:
• Отключите от источника.
• Подождите пять минут, чтобы разрядился заряд.
• Замкните накоротко вводы или, в случае конденсаторов с одним выводом, короткое замыкание между вводом и корпусом.
• Сохраняйте соединение короткого замыкания на месте до тех пор, пока конденсатор не будет подключен для обслуживания.

Об авторе: Джон Мортон, CUSP, начал свою карьеру в электротехнической промышленности в 1970 году в качестве земляка в компании Houston Lighting and Power, ныне известной как CenterPoint Energy. В 1997 году он принял должность инструктора по вопросам электротехники и связи в отделе распространения знаний Техасского университета A&M, а в 2004 году занял свою нынешнюю должность директора по безопасности и обучению в Willbros T&D Services в Техасе.

Как конденсатор работает с переменным током

В предыдущем посте мы видели работу конденсатора с постоянным током. Этот пост о том, как конденсатор работает с переменным током. В электронике переменного тока конденсаторы обрабатывают сигнал переменного тока, чтобы получить выходной сигнал определенной частоты и амплитуды.

Как конденсатор работает с переменным током

Сигнал переменного тока постоянно изменяется во времени. Существуют разные типы сигналов переменного тока e.грамм. синус, треугольник, квадрат и т. д. Давайте сначала проясним две наиболее важные концепции, связанные с конденсатором в цепях переменного тока.

  • Конденсатор пропускает сигнал переменного тока и блокирует постоянный ток.

Это утверждение не на 100% верно. Конденсатор блокирует постоянный ток, за исключением времени, когда он заряжается или разряжается. Конденсатор также в некоторой степени блокирует сигнал переменного тока. Такая природа конденсатора по отношению к сигналу переменного тока обозначается как реактивное сопротивление конденсатора . Реактивное сопротивление работает так же, как сопротивление в цепях постоянного тока.

  • Конденсатор: короткое замыкание на переменный ток и обрыв на постоянный ток.

Вы можете подумать, что если конденсатор пропускает переменный ток намного лучше, чем постоянный, то он должен действовать как короткое замыкание или разрыв цепи с сигналом переменного тока. Ответ — нет. Конденсатор не работает как разомкнутая или короткое замыкание. Тогда возникает вопрос, как конденсатор пропускает переменный ток без короткого замыкания или разрыва цепи. Давайте обсудим…!

По аналогии с водой можно сравнить с работой конденсатора на переменном токе.Рассмотрим пластины конденсатора как два резервуара для воды T1 и T2, заполненные одинаково на половину своей полной емкости. Две трубы, используемые для наполнения / опорожнения резервуаров, действуют как выводы конденсаторов. Между этими двумя трубами используется насос, аналогичный источнику напряжения. Здесь резервуар T1 заполняется отрицательным напряжением и опорожняется положительным напряжением. Танк Т2 работает прямо противоположно Т1. Опорожненный резервуар похож на пластину конденсатора с отрицательным зарядом. Вода, протекающая по трубам, подобна току, протекающему через конденсатор.

Теперь сначала рассмотрим, что наш источник напряжения — постоянный ток. Сигнал постоянного тока постоянный и может быть положительным или отрицательным. При отрицательном напряжении насос будет сливать воду из Т1 и подавать ее в Т2. Через некоторое время поток воды прекращается, Т1 опорожняется, а Т2 полностью заполняется. Следовательно, нет непрерывного потока воды от Т1 к Т2 с постоянным током.

Теперь напряжение заменено на переменное. Сигнал переменного тока постоянно меняется от положительного до отрицательного. Баки опорожняются или заполняются до соответствующих напряжений.Но на этот раз полярность сигнала постоянно меняется с положительной на отрицательную и наоборот. Следовательно, ни Т1, ни Т2 не опорожняются полностью, и вода непрерывно течет в обоих направлениях по трубам. Именно это происходит при работе конденсатора с переменным током. Заряд обкладок конденсатора непрерывно изменяется переменным током. Следовательно, это приводит к потоку электронов через конденсатор.

Емкостное реактивное сопротивление

У резистора и конденсатора есть одно важное сходство.Сопротивление резистора препятствует прохождению тока за счет рассеивания тепла. Способность конденсатора противодействовать протеканию тока (как переменного, так и постоянного) известна как реактивное сопротивление конденсатора. Реактивное сопротивление препятствует прохождению тока без рассеивания тепла. Сопротивление конденсатора току очевидно по своей природе, то есть наблюдается только в какой-то момент. И сопротивление, и реактивное сопротивление измеряются в омах. Термин реактивное сопротивление происходит от того факта, что реакция пластин конденсатора на протекание тока, то есть пластины несут положительный или отрицательный заряд при приложении напряжения к конденсатору.

Частота — важный параметр сигнала переменного тока. Возможно, вы читали, что конденсатор действует как разрыв цепи на низких частотах и ​​короткое замыкание на высоких частотах . Это утверждение основано на том, что частота обратно пропорциональна емкостному сопротивлению . Это колебание напряжения прямо пропорционально току через конденсатор. Чем медленнее колеблется входное напряжение, тем меньше поток электронов через конденсатор и наоборот.

Вспомните математическое представление постоянной времени конденсатора.

Время (τ) = R x C

Следовательно, увеличение емкости увеличивает необходимое время для зарядки, что подразумевает низкую частоту (медленные колебания входного напряжения) и меньший поток электронов через конденсатор.

Математическое выражение емкостного реактивного сопротивления равно

.
Xc = 1 (2πfC). . . . . . . . . . . (Ед. — Ом)
Закон
Ом для емкостного реактивного сопротивления

Как обсуждалось ранее, сопротивление и реактивное сопротивление имеют одну и ту же единицу — Ом.Также существует закон Ома для емкостного реактивного сопротивления. Важное замечание: при применении закона Ома для емкостного реактивного сопротивления частота должна быть постоянной. Давайте выясним, как емкостное реактивное сопротивление изменяется по закону Ома.

Для C = 0,1 мкФ, f = 100 Гц, V = 5 В Для C = 0,1 мкФ, f = 10 кГц, V = 5 В
I = В Xc = 5 (15.91k)

I = 0,314 мА

I = V Xc = 5 (159.15)

I = 31,4 мА

Эти примеры показывают, что изменение частоты входного сигнала изменяет емкостное реактивное сопротивление. Следовательно, частота входного сигнала должна быть постоянной при применении закона Ома для конденсатора с переменным током. На этом пока все. Надеюсь, теперь вы знаете, как конденсатор работает с переменным током. В следующем посте мы обсудим типы конденсаторов. Спасибо, что прочитали, и не забудьте оставить комментарий. В этой серии электроники есть много новых конденсаторов.Продолжайте посещать.

симистор — Какова роль конденсаторов в этой цепи переменного тока?

Для анализа схемы мы можем разбить ее на две части:

1. LC фильтр подавления RFFI

Симистор может использоваться для обеспечения переменного управления мощностью переменного тока с использованием техники «переключения с фазовой задержкой», при которой симистор запускается частично в течение каждого полупериода. Каждый раз, когда триак включается, его ток нагрузки резко (за несколько микросекунд) переключается с нуля на значение, установленное его сопротивлением нагрузки и мгновенными значениями напряжения питания.В резистивно нагруженных схемах это действие переключения неизбежно генерирует импульс RFI, который является наименьшим, когда симистор срабатывает вблизи точек пересечения нуля 0 ° и 180 ° формы сигнала линии питания (в которых токи включения равны минимальный), и является наибольшим, когда устройство срабатывает под углом 90 ° после начала каждого полупериода (когда токи включения максимальны).

Импульсы радиопомех возникают с частотой, вдвое превышающей частоту питающей сети, и могут быть очень раздражающими. Обычно радиочастотные помехи можно устранить, установив на интерфейс простой LC-фильтр.Фильтр устанавливается рядом с симистором и значительно снижает скорость нарастания токов в сети переменного тока.

2. Сеть триггера с переменной задержкой по фазе

Когда на цепь подается входное напряжение, c1 и c2 начинают заряжаться со скоростью, определяемой сопротивлением R2. Каждый раз, когда напряжение на конденсаторе c3 превышает напряжение отключения диакритического контура, диаконтакт срабатывает и начинает проводить. Затем конденсатор C3 начинает разряжаться через проводящий диак в затвор симистора.

Следовательно, симистор включается и пропускает ток на нагрузку. Путем изменения сопротивления R2 изменяется скорость заряда конденсатора и, следовательно, регулируется напряжение, при котором триак запускается как в положительном, так и в отрицательном полупериоде входа.

Отзывы:

Оптимизация симисторов

Снижение шума импульсного регулятора с LC-фильтром

Руководство для начинающих по DIAC

Разница между конденсаторами постоянного и переменного тока

Конденсатор — это электронное устройство, состоящее из двух пластин из электропроводящего материала, разделенных непроводящим материалом или изолятором.Он используется в электрических приложениях и схемах, использующих A / C (переменный ток) и D / C (постоянный ток) для достижения определенных результатов, основанных на способности конденсатора накапливать и разряжать ток.

История конденсаторов

Первым устройством, которое можно назвать конденсатором, является лейденская банка, открытая немецким изобретателем Эвальдом Георгом фон Клейстом в 1745 году. Это была стеклянная банка, частично заполненная водой и закупоренная пробкой, через которую проходил провод. Это.Проволока погружалась в воду и при контакте с источником статического электричества вызывала заряд банки. Яс разряжался, когда провод касался проводящего материала или приближался к нему. В следующем году голландский физик Питер ван Мушенбрук из Лейденского университета независимо открыл тот же принцип емкости.

Типы конденсаторов

Конденсаторы подразделяются на несколько категорий в зависимости от конструкции и материалов, используемых в производстве, но в основном есть два основных типа, что касается электричества: поляризованные и неполяризованные.Поляризованные конденсаторы, известные как электролитические конденсаторы, должны строго соответствовать полярности и напряжению. Неполяризованные конденсаторы имеют ограничения только по номинальному напряжению.

Конденсаторы в цепях постоянного тока

Электролитические конденсаторы на плате

Две обкладки конденсатора, электрически изолированные друг от друга, накапливают энергию в виде емкости. Когда постоянный ток применяется к цепи, имеющей только сопротивление и емкость, конденсатор заряжается до уровня приложенного напряжения.Поскольку постоянный ток течет только в одном направлении, как только конденсатор полностью заряжен, ток больше не течет. Эта характеристика позволяет конденсаторам «блокировать» прохождение постоянного тока.

Конденсаторы в цепях переменного тока

В цепях переменного тока переменный ток периодически меняет направление, заряжая конденсатор в одном направлении, а затем в другом. Поскольку пластины разряжаются во время изменения направления, выходной ток конденсатора чередуется по фазе с переменным напряжением.Именно так конденсаторы «пропускают» переменный ток.

Использование конденсаторов

Конденсаторы находят множество применений в электрических и электронных схемах. Их можно использовать в схемах связи, схемах развязки, схемах фильтрации и в источниках питания для сглаживания выходного сигнала. Конденсаторы находят особое применение в схемах, использующих высокие или низкие частоты, используя присущие конденсаторам характеристики и то, как они реагируют на различные диапазоны частот, демонстрируя различные уровни импеданса.

Противодействие протеканию переменного тока

Противодействие протеканию переменного тока

Есть три фактора, которые могут создать противодействие потоку электронов (току) в цепи переменного тока. Сопротивление, как и сопротивление цепей постоянного тока, измеряется в омах и имеет прямое влияние на переменный ток независимо от частоты. С другой стороны, индуктивное и емкостное сопротивление препятствуют протеканию тока только в цепях переменного тока, а не в цепях постоянного тока. Поскольку переменный ток постоянно меняет направление и интенсивность, катушки индуктивности и конденсаторы также могут препятствовать протеканию тока в цепях переменного тока.Следует также отметить, что индуктивное реактивное сопротивление и емкостное реактивное сопротивление могут создавать фазовый сдвиг между напряжением и током в цепи переменного тока. При анализе цепи переменного тока очень важно учитывать сопротивление, индуктивное реактивное сопротивление и емкостное реактивное сопротивление. Все три влияют на ток в этой цепи.

Сопротивление

Как уже упоминалось, сопротивление создает противодействие току в цепи переменного тока, аналогичное сопротивлению цепи постоянного тока.Ток через резистивную часть цепи переменного тока обратно пропорционален сопротивлению и прямо пропорционален напряжению, приложенному к этой цепи или части цепи. Уравнения I = E / R & E = I × R показывают, как ток связан как с напряжением, так и с сопротивлением. Следует отметить, что сопротивление в цепи переменного тока не создает сдвига фаз между напряжением и током.

Рисунок 9-17. Сопротивление.

На рисунке 9-17 показано, как цепь на 10 Ом пропускает ток 11,5 ампер через резистивную цепь переменного тока на 115 вольт.

Индуктивное реактивное сопротивление

При перемещении магнита через катушку с проволокой на катушке индуцируется напряжение. Если предусмотрена полная цепь, то также будет индуцироваться ток. Величина наведенного напряжения прямо пропорциональна скорости изменения магнитного поля относительно катушки. И наоборот, ток, протекающий через катушку с проволокой, создает магнитное поле. Когда этот провод формируется в катушку, он становится основным индуктором.

Основным действием катушки является ее свойство противодействовать любому изменению тока через нее. Это свойство называется индуктивностью. Когда ток течет через любой проводник, магнитное поле начинает расширяться от центра провода. Когда силовые линии магнитного поля растут наружу через проводник, они индуцируют ЭДС в самом проводнике. Индуцированное напряжение всегда имеет направление, противоположное направлению приложенного тока. Эффекты этой противодействующей ЭДС должны противодействовать приложенному току.Этот эффект — временное состояние. Как только ток в проводнике достигает постоянного значения, силовые линии магнитного поля больше не расширяются, и противодействующая ЭДС больше не присутствует. Поскольку переменный ток постоянно изменяется по величине, индуктивность повторяется в цикле, всегда противоположном приложенному напряжению. Следует отметить, что единицей измерения индуктивности является генри (H).

Физические факторы, влияющие на индуктивность:

  1. Число витков — удвоение числа витков в катушке создает поле в два раза сильнее, если используется тот же ток.Как правило, индуктивность зависит от числа витков в квадрате.
  2. Площадь поперечного сечения катушки — индуктивность катушки увеличивается непосредственно по мере увеличения площади поперечного сечения сердечника. Удвоение радиуса катушки увеличивает индуктивность в четыре раза.
  3. Длина катушки — удвоение длины катушки при сохранении того же числа витков снижает индуктивность наполовину.
  4. Материал сердечника, вокруг которого сформирована катушка — катушки намотаны либо на магнитных, либо на немагнитных материалах.Некоторые немагнитные материалы включают воздух, медь, пластик и стекло. Магнитные материалы включают никель, железо, сталь и кобальт, которые обладают проницаемостью, которая обеспечивает лучший путь для магнитных силовых линий и позволяет создавать более сильное магнитное поле.

Поскольку переменный ток находится в постоянном состоянии изменения, магнитные поля внутри индуктора также непрерывно изменяются и создают наведенное напряжение / ток. Это индуцированное напряжение противостоит приложенному напряжению и называется противо-ЭДС.Это сопротивление называется индуктивным реактивным сопротивлением, обозначается XL и измеряется в омах. Эта характеристика катушки индуктивности также может создавать фазовый сдвиг между напряжением и током в цепи. Фазовый сдвиг, создаваемый индуктивным сопротивлением, всегда приводит к тому, что напряжение ведет к току. То есть напряжение в индуктивной цепи достигает своих пиковых значений до того, как ток достигает пиковых значений.

Индуктивность — это свойство цепи противодействовать любому изменению тока, измеряется в генри.Индуктивное реактивное сопротивление — это мера того, насколько противодействующая ЭДС в цепи противодействует приложенному току. Индуктивное сопротивление компонента прямо пропорционально индуктивности компонента и приложенной к цепи частоте. При увеличении индуктивности или приложенной частоты индуктивное реактивное сопротивление также увеличивается и представляет собой большее сопротивление току в цепи. Это соотношение задается как XL = 2πfL, где XL = индуктивное реактивное сопротивление в омах, L = индуктивность в генри, f = частота в циклах в секунду и π = 3.1416

Рисунок 9-18. Цепь переменного тока, содержащая индуктивность.

На рисунке 9-18 показана последовательная цепь переменного тока, в которой индуктивность составляет 0,146 Генри, а напряжение составляет 110 вольт при частоте 60 циклов в секунду. Индуктивное реактивное сопротивление определяется следующим методом.

X L = 2π × f × L
X L = 6,28 × 60 × 0,146
X L = 55 Ом

В последовательных цепях переменного тока индуктивное реактивное сопротивление добавляется подобно последовательным сопротивлениям в цепи постоянного тока. [Рисунок 9-19] Полное реактивное сопротивление в показанной цепи равно сумме индивидуальных реактивных сопротивлений.

Рисунок 9-19. Последовательные индуктивности.

X L = X L1 + X L2
X L = 10 Ом + 15 Ом
X LT = 25 Ом

Рисунок 9-20. Параллельные индуктивности.

Общее реактивное сопротивление катушек индуктивности, подключенных параллельно, определяется таким же образом, как и полное сопротивление в параллельной цепи. [Рисунок 9-20] Таким образом, полное реактивное сопротивление индуктивностей, соединенных параллельно, как показано, выражается как:

Емкостное реактивное сопротивление

Емкость — это способность тела удерживать электрический заряд.Обычно конденсатор состоит из двух параллельных пластин, разделенных изолятором. Изолятор обычно называют диэлектриком. Пластины конденсатора обладают способностью накапливать электроны при зарядке от источника напряжения. Конденсатор разряжается, когда подаваемое напряжение больше не присутствует, и конденсатор подключается к пути тока. В электрической цепи конденсатор служит резервуаром или хранилищем электроэнергии.

Основной единицей емкости является фарада, обозначаемая буквой F.По определению, один фарад — это один кулон заряда, накопленного с одним вольт на пластинах конденсатора. На практике одна фарада — это большая емкость. Обычно в электронике используются блоки гораздо меньшего размера. Две более распространенные единицы меньшего размера — это микрофарад (мкФ), который составляет 10 -6 фарад, и пикофарад (пФ), который составляет 10 -12 фарад.

Емкость зависит от физических свойств конденсатора:

  1. Емкость параллельных пластин прямо пропорциональна их площади.Большая площадь пластины дает большую емкость, а меньшая площадь дает меньшую емкость. Если удвоить площадь пластин, останется место для вдвое большего заряда.
  2. Емкость параллельных пластин обратно пропорциональна расстоянию между пластинами.
  3. Диэлектрический материал влияет на емкость параллельных пластин. Диэлектрическая проницаемость вакуума определяется как 1, а диэлектрическая проницаемость воздуха очень близка к 1. Эти значения используются в качестве эталона, а все другие материалы имеют значения относительно воздуха (вакуума).

При подаче переменного тока в цепь постоянно меняется заряд на пластинах. [Рис. 9-21] Это означает, что электричество сначала должно течь от Y по часовой стрелке к X, затем от X против часовой стрелки к Y, затем от Y по часовой стрелке к X и так далее. Хотя ток не течет через изолятор между пластинами конденсатора, он постоянно течет в оставшейся части цепи между X и Y. По мере того, как этот ток попеременно поступает в конденсатор и от него, создается определенная задержка по времени.Когда конденсатор заряжается или разряжается через сопротивление, требуется определенное время для полной зарядки или разрядки. Напряжение на конденсаторе не меняется мгновенно. Скорость зарядки или разрядки определяется постоянной времени цепи. Такая скорость заряда и разряда создает противодействие протеканию тока в цепях переменного тока, известное как емкостное реактивное сопротивление. Емкостное реактивное сопротивление обозначается X C и измеряется в омах. Эта характеристика конденсатора также может создавать фазовый сдвиг между напряжением и током в цепи.Фазовый сдвиг, создаваемый емкостным реактивным сопротивлением, всегда приводит к тому, что ток ведет к напряжению. То есть ток емкостной цепи достигает своих пиковых значений до того, как напряжение достигает пиковых значений.

Рисунок 9-21. Конденсатор в цепи переменного тока.

Емкостное реактивное сопротивление — это мера того, насколько емкостная цепь противодействует приложенному току. Емкостное реактивное сопротивление измеряется в омах. Емкостное реактивное сопротивление цепи косвенно пропорционально емкости цепи и приложенной к цепи частоте.При увеличении емкости или приложенной частоты емкостное реактивное сопротивление уменьшается, и наоборот. Это соотношение задается следующим образом:

Где: X C = емкостное реактивное сопротивление в омах, C = емкость в фарадах, f = частота в циклах в секунду и π = 3,1416.

На рисунке 9-21 показана последовательная схема, в которой приложенное напряжение составляет 110 вольт при 400 гц, а емкость конденсатора составляет 80 мФ. Найдите емкостное реактивное сопротивление и ток.

Чтобы найти емкостное реактивное сопротивление, используйте следующее уравнение:

Сначала емкость, 80 мкФ, заменяется на фарады путем деления 80 на 1000000, поскольку 1 миллион микрофарад равен 1 фараду. Это частное равно 0,000080 фарада. Это подставляется в уравнение:

Летный механик рекомендует

Что такое цепь переменного тока? — Различные термины и форма волны

Цепь, возбуждаемая переменным источником, называется цепью переменного тока . Переменный ток (AC) используется в бытовых и промышленных целях. В цепи переменного тока значение величины и направления тока и напряжений непостоянно, оно изменяется через равные промежутки времени.

Он распространяется как синусоидальная волна, завершая один цикл как половина положительного и полуотрицательного цикла и является функцией времени (t) или угла (θ = wt).

В цепи постоянного тока противодействие протеканию тока является единственным сопротивлением цепи, тогда как противодействие протеканию тока в цепи переменного тока происходит из-за сопротивления (R), индуктивного реактивного сопротивления (X L = 2πfL) и емкостное реактивное сопротивление (X C = 1/2 πfC) цепи.

В цепи переменного тока ток и напряжения представлены величиной и направлением . Переменная величина может быть или не совпадать по фазе друг с другом в зависимости от различных параметров схемы, таких как сопротивление, индуктивность и емкость. Синусоидальные переменные величины — это напряжение и ток, которые изменяются согласно синусу угла θ.

Для выработки электроэнергии во всем мире выбираются синусоидальные напряжение и ток по следующим причинам, которые приведены ниже.

  • Синусоидальные напряжение и ток приводят к низким потерям в железе и меди в трансформаторах и вращающихся электрических машинах, что, в свою очередь, повышает эффективность машин переменного тока.
  • Они создают меньше помех для ближайшей системы связи.
  • Они вызывают меньше помех в электрической цепи.

    Переменное напряжение и ток в цепи переменного тока

    Напряжение, полярность и величина которого меняется через равные промежутки времени, называется переменным напряжением .Точно так же направление тока изменяется, и величина тока изменяется со временем, это называется переменным током .

    Когда источник переменного напряжения подключается к сопротивлению нагрузки, как показано на рисунке ниже, ток через него течет в одном направлении, а затем в противоположном, когда полярность меняется на противоположную.

    Схема цепи переменного тока

    Форма волны переменного напряжения в зависимости от времени и тока, протекающего через сопротивление (R) в цепи, показана ниже.

    Существуют различные типы цепей переменного тока, такие как цепь переменного тока, содержащая только сопротивление (R), цепь переменного тока, содержащая только емкость (C), цепь переменного тока, содержащую только индуктивность (L), комбинацию цепи RL, цепь переменного тока, содержащую сопротивление и емкость ( RC), цепь переменного тока, содержащая индуктивность и емкость (LC) и сопротивление, индуктивность и емкость (RLC), цепь переменного тока.

    Различные термины, которые часто используются в цепи переменного тока, следующие:

    Максимальное положительное или отрицательное значение, достигаемое переменной величиной за один полный цикл, называется амплитудой, или пиковым значением, или максимальным значением.Максимальные значения напряжения и тока представлены E m или V m и I m соответственно.

    Половина цикла называется чередованием. Диапазон чередования составляет 180 градусов.

    Когда один набор положительных и отрицательных значений завершается переменным числом или проходит через электрические 360 градусов, считается, что это один полный цикл.

    Значение напряжения или тока в любой момент времени называется мгновенным значением.Обозначается он (i или e).

    Число циклов, выполняемых переменной величиной в секунду, называется частотой. Он измеряется в циклах в секунду (с / с) или в герцах (Гц) и обозначается буквой (f).

    Время в секундах, затрачиваемое напряжением или током для завершения одного цикла, называется периодом времени. Обозначается он (T).

    Форма, полученная путем нанесения мгновенных значений переменной величины, такой как напряжение и ток, вдоль оси y и времени (t) или угла (θ = wt) вдоль оси x, называется формой волны.

Это все о цепях переменного тока.

Phys 3041 Notes — D

Phys 3041 Notes — D

Phys 3041 Заметки

© 1997 Джонатан Карты. Эти примечания не могут быть распространены или дублированы в какой-либо форме, печатной или электронной, без разрешения. Разрешается устанавливать электронные гипертекстовые ссылки только на эти оригинальные документы.

Анализ цепей переменного тока

Цепи переменного тока (переменного тока) — это цепи, в которых напряжения и токи зависят от времени.Они намного богаче по возможностям, чем цепи постоянного тока, и мы рассмотрим здесь цепи переменного тока, содержащие резисторы. конденсаторы и катушки индуктивности. Наш анализ этих схем будет использовать комплексные числа для представления зависящих от времени токов и напряжений. Этот подход заставляет анализ этих цепей следовать линиям, очень похожим на те, что были разработаны для цепей постоянного тока.

Резисторы в цепях переменного тока
Простая цепь переменного тока, которую мы могли бы построить, представляла бы собой зависящий от времени источник напряжения, подключенный к одному резистору.Источник напряжения обеспечивает простое синусоидальное напряжение: В o cos (wt) .

V o называется амплитудой сигнала, а w — угловой частотой. Период сигнала T — это время, необходимое для одного полного цикла. Частота f — это количество циклов, выполняемых за одну секунду (циклов в секунду), а единицей измерения является герц (1 Гц = 1 цикл в секунду). Частота — это просто величина, обратная периоду.

Ток, протекающий в цепи, следует из прямого применения закона Ома и меняет направление через резистор. В этой схеме напряжение и ток «синфазны», оба достигают максимума и минимума одновременно.

Величину переменного напряжения часто характеризуют двумя другими показателями: размахом напряжения, В pp = 2 В o , и среднеквадратичным (среднеквадратичным) напряжением.Среднеквадратичное значение напряжения — это значение, сообщаемое большинством мультиметров при измерении сигналов переменного тока, и такие фразы, как «110 В переменного тока», которые появляются рядом со шнуром питания на бытовых и лабораторных приборах, обычно означают 110 В действующее значение . Для синусоидального напряжения , В среднеквадратического значения = В o /2 -1/2 = 0,707 В o .

Обычно среднеквадратичное значение находится путем возведения напряжения в квадрат, вычисления среднего значения В 2 ( t ) и последующего извлечения квадратного корня.

Среднеквадратичное значение напряжения используется для вычисления средней мощности, рассеиваемой на резисторе. Мгновенная мощность теперь определяется как P (t) = V (t) I (t) , в то время как средняя мощность вычисляется за один полный цикл. С I (t) = V (t) / R , мы имеем P (t) = V 2 (t) / R , и нахождение средней мощности сводится к вычислению среднего квадрата напряжения, как показано выше. Средняя мощность тогда всего

= < V 2 > / R = V rms 2 / R .

Конденсаторы
Конденсаторы — это устройства для хранения заряда. Удобной моделью конденсатора является пара параллельных металлических пластин, каждая из которых имеет площадь A, и разделены расстоянием d . Емкость этой системы представляет собой отношение заряда, накопленного на пластинах (+ Q и -Q), к разнице напряжений, В, , между пластинами:
C = Q / V ,
или емкость можно представить как коэффициент пропорциональности между напряжением и накопленным зарядом:
Q = C V .

Мы будем часто использовать последний в нашем анализе цепей с конденсаторами. Емкость системы параллельных пластин пропорциональна площади пластин (чем больше площадь, тем больше накопленный заряд при фиксированном напряжении) и обратно пропорциональна расстоянию между пластинами. (Электрическое поле между пластинами однородно, поэтому для фиксированного заряда меньшее расстояние означает меньшее напряжение: напомним, что здесь В = E d .) В большинстве случаев емкость увеличивается за счет добавления диэлектрического материала (легко поляризуемого изолятора) между пластинами.Затем емкость увеличивается в раз, равном диэлектрической проницаемости материала.

В цепи постоянного тока (не зависящей от времени) конденсаторы не играют никакой роли, кроме переходной при первом включении питания. После зарядки с ними больше ничего не происходит. Электрические заряды не проходят через конденсатор. Материал между пластинами является изолирующим, и добавление заряда к одной пластине просто отталкивает идентичный заряд от лицевой пластины, оставляя на ней противоположный заряд.Когда конденсатор заряжен, он действует как бесконечное сопротивление. Только когда приложенные напряжения и результирующие токи меняются со временем, конденсаторы могут играть важную роль в работе схемы. Мы рассмотрим переходную характеристику конденсаторов позже.

Рассмотрим конденсатор, подключенный к источнику переменного напряжения.

Поскольку напряжение зависит от времени, заряд на пластинах также должен изменяться со временем:
Q (t) = C V (t) .
Если заряд добавляется или вычитается из пластины, должен быть ток, в данном случае обеспечиваемый источником напряжения:
I = dQ / dt = C dV / dt .
Это поведение сильно отличается от поведения резисторов. Ток пропорционален к скорости изменения напряжения. Чем быстрее изменяется напряжение, тем больше должен быть ток, чтобы обеспечить Q = CV . Для простого синусоидального приложенного напряжения, В (t) = V o cos (wt) , результирующий ток определяется как:
I (t) = -wCV o sin (wt) = wCV o cos (вес + 90) .
Это показывает фазовый сдвиг между напряжением и током в одну четверть цикла.

Когда приложенное напряжение достигает экстремума, ток, протекающий по цепи, равен нулю. И наоборот, всякий раз, когда напряжение проходит через ноль, ток либо максимальный, либо минимальный. В этом случае считается, что ток опережает напряжение: ток достигает максимума на четверть цикла раньше, чем напряжение. (Альтернативное описание напряжения, опережающего ток на 3/4 цикла, одинаково верно, но обычно для определения того, кто впереди, а кто отстает, выбирается меньший из двух углов.)

Эту разность фаз между током и напряжением можно эффективно обработать с помощью комплексных чисел. Для этого реальное управляющее напряжение ( V (t) = V o cos (wt) ) заменяется комплексным управляющим напряжением, полученным сложением мнимого члена:
V (t) = V o. cos (wt) + i V o sin (wt) = V o e iwt .
У этой формы есть два существенных преимущества: мы восстановим нечто похожее на закон Ома; а фазовые сдвиги, которые происходят в более сложных схемах, включающих резисторы и катушки индуктивности вместе с конденсаторами, будут очень естественно выпадать из-за обработки напряжений и токов как комплексных чисел во время вычислений.Все, что нам нужно сделать, это не забыть взять реальную часть наших сложных результатов, чтобы получить реальные результаты.

Применять это к простому источнику переменного напряжения и одиночному конденсатору несложно.
Q (t) = CV o e iwt
I (t) = dQ / dt = C dV / dt
I (t) = iwCV o e iwt
I (t) = iwC V (t).
Обратите внимание, что в этой форме ток теперь пропорционален приложенному напряжению.Это аналог закона Ома для резисторов. Аналогом сопротивления называется импеданс конденсатора:
Z c = V (t) / I (t) = 1 / iwC .
Импеданс — это мнимое число. i содержит информацию о разности фаз 90 o между током и напряжением для конденсаторов. Чтобы найти реальный ток, мы просто расширяем V ( t ) до V o cos (wt) + i V o sin (wt) и умножаем все:
I (t ) = iwCV o cos (wt) + i 2 wCV o sin (wt)
I (t) = -wCV o sin (wt) + iwCV o cos (wt)

и берем только действительную часть результата:
I (t) = -wCV o sin (wt) , это именно то, что мы получали раньше.Для такой простой схемы, как эта, использование сложных представлений для напряжения и тока может показаться чрезмерно сложным, но этот метод окупается возможностью обрабатывать более сложные схемы очень простыми методами.

Обратите внимание, что сопротивление конденсатора зависит от частоты. Импеданс очень велик на низких частотах, и для данной амплитуды V o для управляющего напряжения результирующая амплитуда тока I o будет небольшой.Предельный случай этой ситуации — w = 0 — цепь постоянного тока — где емкость имеет бесконечный импеданс или эффективное сопротивление и ток не течет. Другая крайность возникает на высоких частотах, где полное сопротивление уменьшается до нуля: конденсатор действует как «короткое замыкание» на высоких частотах, где переменный ток может протекать с незначительным падением напряжения на конденсаторе. Величину этого импеданса часто называют реактивным сопротивлением конденсатора: X c = 1 / wC .

alexxlab

Добавить комментарий

Ваш адрес email не будет опубликован. Обязательные поля помечены *